Evaluating $int_0^{pi/2}operatorname{arcsinh}(2tan x)dx$












13












$begingroup$



How to prove $$int_0^{pi/2}operatorname{arcsinh}(2tan x)dx=frac43G+frac13pilnleft(2+sqrt3right),$$where $G$ is Catalan's constant?




I have a premonition that this integral is related to $Imoperatorname{Li}_2left(2pmsqrt3right)$.
Attempt
$$int_0^{pi/2}operatorname{arcsinh}(2tan x)dx\=int_0^inftyfrac{operatorname{arcsinh}(2x)}{1+x^2}dx\
=2int_0^inftyfrac{xcosh x}{4+sinh^2x}dx\
=2int_0^inftyfrac{xcosh x}{3+cosh^2x}dx\
=2int_0^inftysum_{n=0}^infty x(-3)^ncosh^{-2n-1}(x)dx$$

I failed to integrate $xcosh^{-2n-1}(x)$. Mathematica returns a hypergeometric term while integrating it.










share|cite|improve this question









$endgroup$

















    13












    $begingroup$



    How to prove $$int_0^{pi/2}operatorname{arcsinh}(2tan x)dx=frac43G+frac13pilnleft(2+sqrt3right),$$where $G$ is Catalan's constant?




    I have a premonition that this integral is related to $Imoperatorname{Li}_2left(2pmsqrt3right)$.
    Attempt
    $$int_0^{pi/2}operatorname{arcsinh}(2tan x)dx\=int_0^inftyfrac{operatorname{arcsinh}(2x)}{1+x^2}dx\
    =2int_0^inftyfrac{xcosh x}{4+sinh^2x}dx\
    =2int_0^inftyfrac{xcosh x}{3+cosh^2x}dx\
    =2int_0^inftysum_{n=0}^infty x(-3)^ncosh^{-2n-1}(x)dx$$

    I failed to integrate $xcosh^{-2n-1}(x)$. Mathematica returns a hypergeometric term while integrating it.










    share|cite|improve this question









    $endgroup$















      13












      13








      13


      6



      $begingroup$



      How to prove $$int_0^{pi/2}operatorname{arcsinh}(2tan x)dx=frac43G+frac13pilnleft(2+sqrt3right),$$where $G$ is Catalan's constant?




      I have a premonition that this integral is related to $Imoperatorname{Li}_2left(2pmsqrt3right)$.
      Attempt
      $$int_0^{pi/2}operatorname{arcsinh}(2tan x)dx\=int_0^inftyfrac{operatorname{arcsinh}(2x)}{1+x^2}dx\
      =2int_0^inftyfrac{xcosh x}{4+sinh^2x}dx\
      =2int_0^inftyfrac{xcosh x}{3+cosh^2x}dx\
      =2int_0^inftysum_{n=0}^infty x(-3)^ncosh^{-2n-1}(x)dx$$

      I failed to integrate $xcosh^{-2n-1}(x)$. Mathematica returns a hypergeometric term while integrating it.










      share|cite|improve this question









      $endgroup$





      How to prove $$int_0^{pi/2}operatorname{arcsinh}(2tan x)dx=frac43G+frac13pilnleft(2+sqrt3right),$$where $G$ is Catalan's constant?




      I have a premonition that this integral is related to $Imoperatorname{Li}_2left(2pmsqrt3right)$.
      Attempt
      $$int_0^{pi/2}operatorname{arcsinh}(2tan x)dx\=int_0^inftyfrac{operatorname{arcsinh}(2x)}{1+x^2}dx\
      =2int_0^inftyfrac{xcosh x}{4+sinh^2x}dx\
      =2int_0^inftyfrac{xcosh x}{3+cosh^2x}dx\
      =2int_0^inftysum_{n=0}^infty x(-3)^ncosh^{-2n-1}(x)dx$$

      I failed to integrate $xcosh^{-2n-1}(x)$. Mathematica returns a hypergeometric term while integrating it.







      calculus integration definite-integrals polylogarithm






      share|cite|improve this question













      share|cite|improve this question











      share|cite|improve this question




      share|cite|improve this question










      asked Jan 22 at 8:15









      Kemono ChenKemono Chen

      3,1941844




      3,1941844






















          6 Answers
          6






          active

          oldest

          votes


















          10












          $begingroup$

          I will begin with the same approach as in @DavidG's answer, but will settle in a different argument. Let



          $$ J(t) = int_{0}^{infty} frac{operatorname{arsinh}(tx)}{1+x^2} , mathrm{d}x. $$



          Our goal is to compute $J(2)$ using Feynman's trick. Differentiating $J(t)$ and substituting $x=sqrt{u^{-2}-1}$, we obtain



          begin{align*}
          J'(t)
          = int_{0}^{infty} frac{x}{(1+x^2)sqrt{1+t^2x^2}} , mathrm{d}x
          = int_{0}^{1} frac{1}{sqrt{1 - (1-t^2)(1-u^2)}} , mathrm{d}u.
          end{align*}



          So it follows that



          begin{align*}
          J(2)
          &= int_{0}^{2} int_{0}^{1} frac{1}{sqrt{1 - (1-t^2)(1-u^2)}} , mathrm{d}umathrm{d}t \
          &= int_{0}^{1} int_{0}^{1} frac{1}{sqrt{1 - (1-t^2)(1-u^2)}} , mathrm{d}umathrm{d}t
          + int_{1}^{2} int_{0}^{1} frac{1}{sqrt{1 + (t^2-1)(1-u^2)}} , mathrm{d}umathrm{d}t.
          end{align*}



          The inner integral is easily computed, yielding



          begin{align*}
          J(2)
          &= int_{0}^{1} frac{operatorname{artanh}left( sqrt{1 - t^2} right)}{sqrt{1 - t^2}} , mathrm{d}t
          + int_{1}^{2} frac{arctanleft(sqrt{t^2-1}right)}{sqrt{t^2 - 1}} , mathrm{d}t.
          end{align*}



          Now we substitute $t = operatorname{sech} varphi$ for the first integral and $t = sec theta$ for the second integral. This yields



          begin{align*}
          J(2)
          &= int_{0}^{infty} frac{varphi}{coshvarphi} , mathrm{d}varphi
          + int_{0}^{frac{pi}{3}} frac{theta}{costheta} , mathrm{d}theta.
          end{align*}



          These integrals can be computed as follows:





          • Using $ operatorname{sech}varphi = frac{2e^{-varphi}}{1 + e^{-2varphi}} = 2 sum_{n=0}^{infty} (-1)^n e^{-(2n+1)varphi} $, we obtain



            $$ int_{0}^{infty} frac{varphi}{coshvarphi} , mathrm{d}varphi
            = 2 sum_{n=0}^{infty} (-1)^n int_{0}^{infty} varphi e^{-(2n+1)varphi} , mathrm{d}varphi
            = 2 sum_{n=0}^{infty} frac{(-1)^n}{(2n+1)^2}
            = 2G. $$




          • Taking integration by parts,



            begin{align*}
            int_{0}^{frac{pi}{3}} frac{theta}{costheta} , mathrm{d}theta
            &= left[ - theta log left( tan left( frac{pi}{4} - frac{theta}{2} right) right) right]_{0}^{frac{pi}{3}} + int_{0}^{frac{pi}{3}} log left( tan left( frac{pi}{4} - frac{theta}{2} right) right) , mathrm{d}theta \
            &= frac{pi}{3} log left( 2 + sqrt{3} right) + 2 int_{frac{pi}{12}}^{frac{pi}{4}} log left( tan theta right) , mathrm{d}theta.
            end{align*}



            This can be computed by using the Fourier series $log left( tan theta right) = - 2 sum_{n=0}^{infty} frac{cos(4n+2)theta}{2n+1} $ to yield



            begin{align*}
            int_{0}^{frac{pi}{3}} frac{theta}{costheta} , mathrm{d}theta
            &= frac{pi}{3} log left( 2 + sqrt{3} right) - 2 sum_{n=0}^{infty} frac{sinleft( frac{pi}{2} (2n+1) right) - sinleft( frac{pi}{6} (2n+1) right)}{(2n+1)^2} \
            &= frac{pi}{3} log left( 2 + sqrt{3} right) - frac{2}{3}G.
            end{align*}




          Combining two result, we obtain the desired answer.






          share|cite|improve this answer









          $endgroup$









          • 1




            $begingroup$
            This is beauty !
            $endgroup$
            – Claude Leibovici
            Jan 22 at 16:10



















          10












          $begingroup$

          On the path of Kemono Chen...



          begin{align}J&=int_0^{pi/2}operatorname{arcsinh}(2tan x)dxend{align}



          Perform the change of variable $y=operatorname{arcsinh}(2tan x)$,



          begin{align}J&=int_0^{+infty}frac{2xcosh x}{4+sinh^2 x},dx\
          &=int_0^{+infty}frac{4xleft(text{e}^{x}+text{e}^{-x}right)}{14+text{e}^{2x}+text{e}^{-2x}},dx\
          &=int_0^{+infty}frac{4xtext{e}^{-x}left(text{e}^{2x}+1right)}{14+text{e}^{2x}+text{e}^{-2x}},dx\
          end{align}



          Perform the change of variable $y=text{e}^{-x}$,



          begin{align}J&=-int_0^1 frac{4ln xleft(1+frac{1}{x^2}right)}{14+x^2+frac{1}{x^2}}\
          &=-int_0^1 frac{4ln xleft(1+x^2right)}{x^4+14x^2+1}\
          &=left[-arctanleft(frac{4x}{1-x^2}right)ln xright]_0^1+int_0^1 frac{arctanleft(frac{4x}{1-x^2}right)}{x},dx\
          &=int_0^1 frac{arctanleft(frac{4x}{1-x^2}right)}{x},dx\
          &=int_0^1 frac{arctanleft(left(2+sqrt{3}right)xright)}{x},dx+int_0^1 frac{arctanleft(left(2-sqrt{3}right)xright)}{x},dx\
          end{align}



          In the first integral perform the change of variable $y=left(2+sqrt{3}right)x$,



          In the second integral perform the change of variable $y=left(2-sqrt{3}right)x$,



          begin{align}J&=int_0^{2+sqrt{3}}frac{arctan x}{x},dx+int_0^{2-sqrt{3}}frac{arctan x}{x},dx\
          &=Big[arctan xln xBig]_0^{2+sqrt{3}}-int_0^{2+sqrt{3}}frac{ln x}{1+x^2},dx+Big[arctan xln xBig]_0^{2-sqrt{3}}-int_0^{2-sqrt{3}}frac{ln x}{1+x^2},dx\
          &=frac{5pi}{12}lnleft(2+sqrt{3}right)-int_0^{2+sqrt{3}}frac{ln x}{1+x^2},dx+frac{pi}{12}lnleft(2-sqrt{3}right)-int_0^{2-sqrt{3}}frac{ln x}{1+x^2},dx\
          &=frac{pi}{3}lnleft(2+sqrt{3}right)-int_0^{2+sqrt{3}}frac{ln x}{1+x^2},dx-int_0^{2-sqrt{3}}frac{ln x}{1+x^2},dx
          end{align}



          In the first integral perform the change of variable $y=dfrac{1}{x}$,



          begin{align}J&=frac{pi}{3}lnleft(2+sqrt{3}right)+int_{2-sqrt{3}}^{+infty}frac{ln x}{1+x^2},dx-int_0^{2-sqrt{3}}frac{ln x}{1+x^2},dx\
          &=frac{pi}{3}lnleft(2+sqrt{3}right)+int_0^{+infty}frac{ln x}{1+x^2},dx-2int_0^{2-sqrt{3}}frac{ln x}{1+x^2},dx\
          &=frac{pi}{3}lnleft(2+sqrt{3}right)-2int_0^{2-sqrt{3}}frac{ln x}{1+x^2},dx\
          end{align}



          Perform the change of variable $y=tan x$,



          begin{align}J&=frac{pi}{3}lnleft(2+sqrt{3}right)-2int_0^{frac{pi}{12}}lnleft(tan xright),dx\
          end{align}



          It is well known that,



          begin{align}
          int_0^{frac{pi}{12}}lnleft(tan xright),dx=-frac{2}{3}text{G}
          end{align}



          (see: Integral: $int_0^{pi/12} ln(tan x),dx$ )



          Thus,



          begin{align}J&=frac{pi}{3}lnleft(2+sqrt{3}right)-2times -frac{2}{3}text{G}\
          &=boxed{frac{pi}{3}lnleft(2+sqrt{3}right)+frac{4}{3}text{G}}
          end{align}



          NB:



          Observe that,



          begin{align}2-sqrt{3}&=frac{1}{2+sqrt{3}}\
          lnleft(2-sqrt{3}right)&=-lnleft(2+sqrt{3}right)\
          int_0^infty frac{ln x}{1+x^2},dx&=0
          end{align}

          (perform the change of variable $y=dfrac{1}{x}$ )






          share|cite|improve this answer











          $endgroup$













          • $begingroup$
            This is beauty !
            $endgroup$
            – Claude Leibovici
            Jan 22 at 16:10










          • $begingroup$
            How did you do $$intarctanfrac{4x}{1-x^2}frac{mathrm dx}{x}=intarctan(2+sqrt3)xfrac{mathrm dx}{x}+intarctan(2-sqrt3)xfrac{mathrm dx}{x}$$
            $endgroup$
            – clathratus
            Jan 22 at 16:10








          • 1




            $begingroup$
            @Clathratus: if $ab<1$then begin{align}arctan a+arctan b=arctanleft(frac{a+b}{1-ab}right)end{align}
            $endgroup$
            – FDP
            Jan 22 at 16:23



















          4












          $begingroup$

          Here is another variation along a theme which like @DavidG approach uses Feynman's trick of differentiating under the integral sign.



          Let
          $$I(a) = int_0^{frac{pi}{2}} operatorname{arcsinh} (a tan x) , dx, qquad a > 1.$$
          We are required to find $I(2)$. We start by finding $I(1)$ first, which will be needed later on.



          For $a = 1$ we have
          begin{align}
          I(1) &= int_0^{frac{pi}{2}} operatorname{arcsinh} (tan x) , dx\
          &= int_0^{frac{pi}{2}} ln left (frac{1 + sin x}{cos x} right ) , dx\
          &= int_0^{frac{pi}{2}} ln (1 + sin x) , dx - int_0^{frac{pi}{2}} ln (cos x) , dx.
          end{align}

          Now the first of these integrals can be found, for example, by rewriting it as
          begin{align}
          int_0^{frac{pi}{2}} ln (1 + sin x) , dx &= int_0^{frac{pi}{2}} ln (1 + cos x) , dx\
          &= int_0^{frac{pi}{2}} ln left (frac{1}{2} cos^2 frac{x}{2} right ) , dx\
          &= frac{pi}{2} ln 2 + 4 int_0^{frac{pi}{4}} ln (cos x) , dx
          end{align}

          and then employing the Fourier series representation for $ln (cos x)$ found here. The final result is
          $$int_0^{frac{pi}{2}} ln (1 + sin x) , dx = 2 mathbf{G} - frac{pi}{2} ln 2.$$
          Here $mathbf{G}$ is Catalan's constant.



          The second of the integrals is very well known. Here
          $$int_0^{frac{pi}{2}} ln (cos x) , dx = - frac{pi}{2} ln 2.$$
          Thus $I(1) = 2 mathbf{G}$.



          Moving to the main event, by differentiating under the integral sign with respect to $a$ we have
          begin{align}
          I'(a) &= int_0^{frac{pi}{2}} frac{tan x}{sqrt{a^2 tan^2 x + 1}} , dx\
          &= int_0^{frac{pi}{2}} frac{sin x}{sqrt{a^2 - (a^2 - 1) cos^2 x}} , dx.
          end{align}

          Observe the term $(a^2 - 1)$ is positive since $a > 1$. Letting $u = cos x$ one has
          $$I'(a) = int_0^1 frac{du}{sqrt{a^2 - (a^2 - 1) u^2}} = frac{1}{sqrt{a^2 - 1}} sin^{-1} left (frac{sqrt{a^2 - 1}}{a} right ).$$



          As we require $I(2)$ observe that
          $$I(2) - I(1) = int_1^2 I'(a) , da.$$
          Thus
          $$I(2) = 2 mathbf{G} + int_1^2 frac{1}{sqrt{a^2 - 1}} sin^{-1} left (frac{sqrt{a^2 - 1}}{a} right ) , da.$$
          Integrating by parts leads to
          $$I(2) = 2 mathbf{G} + frac{pi}{3} ln (2 + sqrt{3}) - int_1^2 frac{cosh^{-1} a}{a sqrt{a^2 - 1}} , da.$$
          Now let $a = cosh t$. This gives
          $$I(2) = 2mathbf{G} + frac{pi}{3} ln (2 + sqrt{3}) - int_0^{ln(2 + sqrt{3})} frac{t}{cosh t} , dt.$$
          Then let $t = ln y$. This gives
          $$I(2) = 2mathbf{G} + frac{pi}{3} ln (2 + sqrt{3}) - 2int_1^{2 + sqrt{3}} frac{ln y}{1 + y^2} , dy.$$
          Now let $y = tan theta$. Then we have
          $$I(2) = 2mathbf{G} + frac{pi}{3} ln (2 + sqrt{3}) - 2int_{frac{pi}{4}}^{frac{5pi}{12}} ln (tan theta) , dtheta.$$
          Finally, enforcing a substitution of $theta mapsto dfrac{pi}{2} - theta$ leads to
          begin{align}
          I(2) &= 2mathbf{G} + frac{pi}{3} ln (2 + sqrt{3}) + 2 int_{frac{pi}{12}}^{frac{pi}{4}} ln (tan theta) , dtheta\
          &= 2mathbf{G} + frac{pi}{3} ln (2 + sqrt{3}) + 2int_0^{frac{pi}{4}} ln (tan theta) , dtheta - 2 int_0^{frac{pi}{12}} ln (tan theta) , dtheta.
          end{align}

          For the first of the integrals we have
          $$int_0^{frac{pi}{4}} ln (tan x) , dx = -mathbf{G}.$$
          While for the second of the integrals we have
          $$int_0^{frac{pi}{12}} ln (tan x) , dx = -frac{2}{3} mathbf{G}.$$
          So finally
          $$I(2) = 2mathbf{G} + frac{pi}{3} ln (2 + sqrt{3}) - 2mathbf{G} + frac{4}{3} mathbf{G},$$
          or
          $$int_0^{frac{pi}{2}} operatorname{arcsinh} (2 tan x) , dx = frac{pi}{3} ln (2 + sqrt{3}) + frac{4}{3} mathbf{G},$$
          as announced.






          share|cite|improve this answer









          $endgroup$









          • 1




            $begingroup$
            begin{align}A&=int_0^{frac{pi}{4}}ln(cos x),dx\ B&=int_0^{frac{pi}{4}}ln(sin x),dx\ B-A&=int_0^{frac{pi}{4}}ln(tan x),dx\ &=-text{G}\ A+B&=int_0^{frac{pi}{4}}ln(sin xcos x),dx\ &=int_0^{frac{pi}{4}}lnleft(frac{sin(2x)}{2}right),dx\ &=frac{1}{2}int_0^{frac{pi}{2}}lnleft(frac{sin x}{2}right),dx\ &=frac{1}{2}int_0^{frac{pi}{2}}lnleft(sin xright),dx-frac{pi}{4}ln 2\ &=-frac{1}{2}piln 2\ A&=frac{1}{2}left(A+B-left(B-Aright)right)\ &=frac{1}{2}text{G}-frac{1}{4}piln 2 end{align}
            $endgroup$
            – FDP
            Jan 23 at 13:44










          • $begingroup$
            Yes, very nice.
            $endgroup$
            – omegadot
            Jan 24 at 1:29



















          3












          $begingroup$

          Sorry in a rush, so only a PARTIAL SOLUTION:



          Here I will employ Feynman's Trick:
          begin{equation}
          I = int_0^{infty}frac{operatorname{arcsinh(2x)}}{1 + x^2}:dx
          end{equation}



          Let
          begin{equation}
          J(t) = int_0^{infty}frac{operatorname{arcsinh(tx)}}{1 + x^2}:dx
          end{equation}



          We observe that $J(2) = I$ and $J(0) = 0$. Using Leibniz's Integral rule we differentiate with respect to '$t$':



          begin{equation}
          J'(t) = int_0^{infty}frac{x}{sqrt{1 + t^2x^2}}frac{1}{1 + x^2}:dx = left[frac{1}{sqrt{t^2 - 1}} cdot arctanleft(sqrt{frac{1 + t^2x^2}{t^2 - 1}} right) right]_0^{infty} = frac{1}{sqrt{t^2 - 1}}left[frac{pi}{2} - arctanleft(frac{1}{sqrt{t^2 -1}} right) right]
          end{equation}



          Thus,



          begin{align}
          J(t) &= int frac{1}{sqrt{t^2 - 1}}left[frac{pi}{2} - arctanleft(frac{x}{sqrt{t^2 -1}} right) right]:dt \
          &= int frac{pi}{2}cdot frac{1}{sqrt{t^2 - 1}}:dt - int frac{1}{sqrt{t^2 - 1}}arctanleft(frac{1}{sqrt{t^2 -1}} right):dt = I_1 - I_2
          end{align}



          For $I_1$:



          begin{equation}
          I_1 = int frac{pi}{2}cdot frac{1}{sqrt{t^2 - 1}}:dt = frac{pi}{2}lnleft| sqrt{t^2 - 1} + tright| + C_1
          end{equation}



          Where $C_1$ is the constant of integration. Note that $I_1(2) = lnleft|2 + sqrt{3} right|$



          For $I_2$:



          begin{equation}
          I_2 = int frac{1}{sqrt{t^2 - 1}}arctanleft(frac{1}{sqrt{t^2 -1}} right):dt
          end{equation}



          Unfortunately this is not so easy to evaluate. I will first make the substitution $u = frac{1}{sqrt{t^2 - 1}}$:



          begin{align}
          I_2 &= int frac{1}{sqrt{t^2 - 1}}arctanleft(frac{1}{sqrt{t^2 -1}} right):dt = int u cdot arctan(u) cdot frac{-u^4}{sqrt{1 +u^2}}:du\
          & = - int frac{-u^5}{sqrt{1 +u^2}} cdot arctan(u) :du
          end{align}






          share|cite|improve this answer











          $endgroup$













          • $begingroup$
            Will post up full solution later.
            $endgroup$
            – DavidG
            Jan 22 at 10:08



















          3












          $begingroup$

          $$
          begin{align}newcommand{arcsinh}{operatorname{arcsinh}}
          int_0^inftyfrac{arcsinh(x)}{1+x^2},mathrm{d}x
          &=int_0^inftyfrac{x,mathrm{d}x}{cosh(x)}tag{1a}\
          &=int_0^inftyfrac{2x,mathrm{d}x}{e^x+e^{-x}}tag{1b}\
          &=2sum_{k=0}^infty(-1)^kint_0^infty x,e^{-(2k+1)x},mathrm{d}xtag{1c}\ &=2sum_{k=0}^inftyfrac{(-1)^k}{(2k+1)^2}tag{1d}\[6pt]
          &=2mathrm{G}tag{1e}
          end{align}
          $$

          Explanation:
          $text{(1a)}$: substitute $xmapstosinh(x)$
          $text{(1b)}$: write $cosh(x)$ as exponentials
          $text{(1c)}$: expand $left(e^x+e^{-x}right)^{-1}$ as a power series
          $text{(1d)}$: evaluate the integral
          $text{(1e)}$: use the definition of Catalan's Constant
          $$
          begin{align}
          frac{mathrm{d}}{mathrm{d}a}int_0^inftyfrac{arcsinh(ax)}{1+x^2},mathrm{d}x
          &=int_0^inftyfrac{x}{1+x^2}frac{mathrm{d}x}{sqrt{1+a^2x^2}}tag{2a}\
          &=int_0^inftyfrac{x}{a^2+x^2}frac{mathrm{d}x}{sqrt{1+x^2}}tag{2b}\
          &=int_0^{pi/2}frac{mathrm{d}sec(x)}{a^2+tan^2(x)}tag{2c}\
          &=int_1^inftyfrac{mathrm{d}x}{x^2+left(a^2-1right)}tag{2d}\
          &=frac{arctanleft(sqrt{a^2-1}right)}{sqrt{a^2-1}}tag{2e}
          end{align}
          $$

          Explanation:
          $text{(2a)}$: $frac{mathrm{d}}{mathrm{d}a}arcsinh(ax)=frac{x}{sqrt{1+a^2x^2}}$
          $text{(2b)}$: substitute $xmapsto x/a$
          $text{(2c)}$: substitute $xmapstotan(x)$
          $text{(2d)}$: substitute $sec(x)mapsto x$
          $text{(2e)}$: arctan integral



          Therefore,
          $$
          begin{align}newcommand{Li}{operatorname{Li}}
          int_0^inftyfrac{arcsinh(2x)}{1+x^2},mathrm{d}x
          &=2mathrm{G}+int_1^2frac{arctanleft(sqrt{a^2-1}right)}{sqrt{a^2-1}},mathrm{d}atag{3a}\
          &=2mathrm{G}+int_0^{pi/3}frac{a}{cos(a)},mathrm{d}atag{3b}\
          &=2mathrm{G}+2int_0^{pi/3}frac{a}{e^{ia}+e^{-ia}},mathrm{d}atag{3c}\
          &=2mathrm{G}+2sum_{k=0}^infty(-1)^kint_0^{pi/3}ae^{-i(2k+1)a},mathrm{d}atag{3d}\
          &=2mathrm{G}+2sum_{k=0}^inftyfrac{(-1)^k}{(2k+1)^2}int_0^{(2k+1)pi/3}ae^{-ia},mathrm{d}atag{3e}\
          &=2mathrm{G}+2sum_{k=0}^inftyfrac{(-1)^k}{(2k+1)^2}left[(1+i(2k+1)pi/3)e^{-i(2k+1)pi/3}-1right]tag{3f}\
          &=2mathrm{G}+fracpi3logleft(frac{2+sqrt3+i}{2-sqrt3-i}right)+2sum_{k=0}^inftyfrac{(-1)^k}{(2k+1)^2}left[e^{-i(2k+1)pi/3}-1right]tag{3g}\
          &=2mathrm{G}+fracpi3logleft(left(2+sqrt3right)iright)
          +sum_{k=0}^infty(-1)^kscriptsizeleft[frac{-1-isqrt3}{(6k+1)^2}-frac{-4}{(6k+3)^2}+frac{-1+isqrt3}{(6k+5)^2}right]tag{3h}\
          &=frac43mathrm{G}+fracpi3logleft(2+sqrt3right)+ileft[frac{pi^2}6-sqrt3sum_{k=0}^infty(-1)^kscriptsizeleft(frac1{(6k+1)^2}-frac1{(6k+5)^2}right)right]tag{3i}\
          &=frac43mathrm{G}+fracpi3logleft(2+sqrt3right)+ileft[frac{pi^2}6-frac{sqrt3}{36}sum_{kinmathbb{Z}}frac{(-1)^k}{left(k+frac16right)^2}right]tag{3j}\
          &=bbox[5px,border:2px solid #C0A000]{frac43mathrm{G}+fracpi3logleft(2+sqrt3right)}tag{3k}
          end{align}
          $$

          Explanation:
          $text{(3a)}$: apply $(1)$ and $(2)$
          $text{(3b)}$: substitute $amapstosec(a)$
          $text{(3c)}$: write $cos(x)$ as exponentials
          $text{(3d)}$: expand $left(e^{ix}+e^{-ix}right)^{-1}$ as a power series
          $text{(3e)}$: substitute $amapsto a/(2k+1)$
          $text{(3f)}$: integrate
          $text{(3g)}$: recognize the series for $arctan(x)=frac1{2i}logleft(frac{1+ix}{1-ix}right)$
          $text{(3h)}$: evaluate the exponentials
          $text{(3i)}$: separate the real and imaginary parts
          $text{(3j)}$: rewrite the sum
          $text{(3k)}$: $sum_{kinmathbb{Z}}frac{(-1)^k}{(k+x)^2}=pi^2cot(pi x)csc(pi x)$






          share|cite|improve this answer











          $endgroup$













          • $begingroup$
            Seems like there should be a ^2 in the denominator of the sum of the explanation (3k)
            $endgroup$
            – Kemono Chen
            Jan 24 at 9:03










          • $begingroup$
            @KemonoChen: Indeed. I put the parentheses there because I meant to square the denominator. Thanks. While it's true that we can just take the real part of the right side since the quantity is real, I wanted to make sure I hadn't made a mistake and verify that the imaginary part was $0$.
            $endgroup$
            – robjohn
            Jan 24 at 9:25





















          1












          $begingroup$

          This is not an answer.



          Your premonition seems to be good. Using another CAS,
          $$2int_0^inftyfrac{xcosh( x)}{3+cosh^2(x)},dx=-frac{pi}{4} log left(7-4 sqrt{3}right)-i left(text{Li}_2left(-i left(-2+sqrt{3}right)right)-text{Li}_2left(i
          left(-2+sqrt{3}right)right)right)$$
          Now, ???






          share|cite|improve this answer









          $endgroup$













            Your Answer





            StackExchange.ifUsing("editor", function () {
            return StackExchange.using("mathjaxEditing", function () {
            StackExchange.MarkdownEditor.creationCallbacks.add(function (editor, postfix) {
            StackExchange.mathjaxEditing.prepareWmdForMathJax(editor, postfix, [["$", "$"], ["\\(","\\)"]]);
            });
            });
            }, "mathjax-editing");

            StackExchange.ready(function() {
            var channelOptions = {
            tags: "".split(" "),
            id: "69"
            };
            initTagRenderer("".split(" "), "".split(" "), channelOptions);

            StackExchange.using("externalEditor", function() {
            // Have to fire editor after snippets, if snippets enabled
            if (StackExchange.settings.snippets.snippetsEnabled) {
            StackExchange.using("snippets", function() {
            createEditor();
            });
            }
            else {
            createEditor();
            }
            });

            function createEditor() {
            StackExchange.prepareEditor({
            heartbeatType: 'answer',
            autoActivateHeartbeat: false,
            convertImagesToLinks: true,
            noModals: true,
            showLowRepImageUploadWarning: true,
            reputationToPostImages: 10,
            bindNavPrevention: true,
            postfix: "",
            imageUploader: {
            brandingHtml: "Powered by u003ca class="icon-imgur-white" href="https://imgur.com/"u003eu003c/au003e",
            contentPolicyHtml: "User contributions licensed under u003ca href="https://creativecommons.org/licenses/by-sa/3.0/"u003ecc by-sa 3.0 with attribution requiredu003c/au003e u003ca href="https://stackoverflow.com/legal/content-policy"u003e(content policy)u003c/au003e",
            allowUrls: true
            },
            noCode: true, onDemand: true,
            discardSelector: ".discard-answer"
            ,immediatelyShowMarkdownHelp:true
            });


            }
            });














            draft saved

            draft discarded


















            StackExchange.ready(
            function () {
            StackExchange.openid.initPostLogin('.new-post-login', 'https%3a%2f%2fmath.stackexchange.com%2fquestions%2f3082875%2fevaluating-int-0-pi-2-operatornamearcsinh2-tan-xdx%23new-answer', 'question_page');
            }
            );

            Post as a guest















            Required, but never shown

























            6 Answers
            6






            active

            oldest

            votes








            6 Answers
            6






            active

            oldest

            votes









            active

            oldest

            votes






            active

            oldest

            votes









            10












            $begingroup$

            I will begin with the same approach as in @DavidG's answer, but will settle in a different argument. Let



            $$ J(t) = int_{0}^{infty} frac{operatorname{arsinh}(tx)}{1+x^2} , mathrm{d}x. $$



            Our goal is to compute $J(2)$ using Feynman's trick. Differentiating $J(t)$ and substituting $x=sqrt{u^{-2}-1}$, we obtain



            begin{align*}
            J'(t)
            = int_{0}^{infty} frac{x}{(1+x^2)sqrt{1+t^2x^2}} , mathrm{d}x
            = int_{0}^{1} frac{1}{sqrt{1 - (1-t^2)(1-u^2)}} , mathrm{d}u.
            end{align*}



            So it follows that



            begin{align*}
            J(2)
            &= int_{0}^{2} int_{0}^{1} frac{1}{sqrt{1 - (1-t^2)(1-u^2)}} , mathrm{d}umathrm{d}t \
            &= int_{0}^{1} int_{0}^{1} frac{1}{sqrt{1 - (1-t^2)(1-u^2)}} , mathrm{d}umathrm{d}t
            + int_{1}^{2} int_{0}^{1} frac{1}{sqrt{1 + (t^2-1)(1-u^2)}} , mathrm{d}umathrm{d}t.
            end{align*}



            The inner integral is easily computed, yielding



            begin{align*}
            J(2)
            &= int_{0}^{1} frac{operatorname{artanh}left( sqrt{1 - t^2} right)}{sqrt{1 - t^2}} , mathrm{d}t
            + int_{1}^{2} frac{arctanleft(sqrt{t^2-1}right)}{sqrt{t^2 - 1}} , mathrm{d}t.
            end{align*}



            Now we substitute $t = operatorname{sech} varphi$ for the first integral and $t = sec theta$ for the second integral. This yields



            begin{align*}
            J(2)
            &= int_{0}^{infty} frac{varphi}{coshvarphi} , mathrm{d}varphi
            + int_{0}^{frac{pi}{3}} frac{theta}{costheta} , mathrm{d}theta.
            end{align*}



            These integrals can be computed as follows:





            • Using $ operatorname{sech}varphi = frac{2e^{-varphi}}{1 + e^{-2varphi}} = 2 sum_{n=0}^{infty} (-1)^n e^{-(2n+1)varphi} $, we obtain



              $$ int_{0}^{infty} frac{varphi}{coshvarphi} , mathrm{d}varphi
              = 2 sum_{n=0}^{infty} (-1)^n int_{0}^{infty} varphi e^{-(2n+1)varphi} , mathrm{d}varphi
              = 2 sum_{n=0}^{infty} frac{(-1)^n}{(2n+1)^2}
              = 2G. $$




            • Taking integration by parts,



              begin{align*}
              int_{0}^{frac{pi}{3}} frac{theta}{costheta} , mathrm{d}theta
              &= left[ - theta log left( tan left( frac{pi}{4} - frac{theta}{2} right) right) right]_{0}^{frac{pi}{3}} + int_{0}^{frac{pi}{3}} log left( tan left( frac{pi}{4} - frac{theta}{2} right) right) , mathrm{d}theta \
              &= frac{pi}{3} log left( 2 + sqrt{3} right) + 2 int_{frac{pi}{12}}^{frac{pi}{4}} log left( tan theta right) , mathrm{d}theta.
              end{align*}



              This can be computed by using the Fourier series $log left( tan theta right) = - 2 sum_{n=0}^{infty} frac{cos(4n+2)theta}{2n+1} $ to yield



              begin{align*}
              int_{0}^{frac{pi}{3}} frac{theta}{costheta} , mathrm{d}theta
              &= frac{pi}{3} log left( 2 + sqrt{3} right) - 2 sum_{n=0}^{infty} frac{sinleft( frac{pi}{2} (2n+1) right) - sinleft( frac{pi}{6} (2n+1) right)}{(2n+1)^2} \
              &= frac{pi}{3} log left( 2 + sqrt{3} right) - frac{2}{3}G.
              end{align*}




            Combining two result, we obtain the desired answer.






            share|cite|improve this answer









            $endgroup$









            • 1




              $begingroup$
              This is beauty !
              $endgroup$
              – Claude Leibovici
              Jan 22 at 16:10
















            10












            $begingroup$

            I will begin with the same approach as in @DavidG's answer, but will settle in a different argument. Let



            $$ J(t) = int_{0}^{infty} frac{operatorname{arsinh}(tx)}{1+x^2} , mathrm{d}x. $$



            Our goal is to compute $J(2)$ using Feynman's trick. Differentiating $J(t)$ and substituting $x=sqrt{u^{-2}-1}$, we obtain



            begin{align*}
            J'(t)
            = int_{0}^{infty} frac{x}{(1+x^2)sqrt{1+t^2x^2}} , mathrm{d}x
            = int_{0}^{1} frac{1}{sqrt{1 - (1-t^2)(1-u^2)}} , mathrm{d}u.
            end{align*}



            So it follows that



            begin{align*}
            J(2)
            &= int_{0}^{2} int_{0}^{1} frac{1}{sqrt{1 - (1-t^2)(1-u^2)}} , mathrm{d}umathrm{d}t \
            &= int_{0}^{1} int_{0}^{1} frac{1}{sqrt{1 - (1-t^2)(1-u^2)}} , mathrm{d}umathrm{d}t
            + int_{1}^{2} int_{0}^{1} frac{1}{sqrt{1 + (t^2-1)(1-u^2)}} , mathrm{d}umathrm{d}t.
            end{align*}



            The inner integral is easily computed, yielding



            begin{align*}
            J(2)
            &= int_{0}^{1} frac{operatorname{artanh}left( sqrt{1 - t^2} right)}{sqrt{1 - t^2}} , mathrm{d}t
            + int_{1}^{2} frac{arctanleft(sqrt{t^2-1}right)}{sqrt{t^2 - 1}} , mathrm{d}t.
            end{align*}



            Now we substitute $t = operatorname{sech} varphi$ for the first integral and $t = sec theta$ for the second integral. This yields



            begin{align*}
            J(2)
            &= int_{0}^{infty} frac{varphi}{coshvarphi} , mathrm{d}varphi
            + int_{0}^{frac{pi}{3}} frac{theta}{costheta} , mathrm{d}theta.
            end{align*}



            These integrals can be computed as follows:





            • Using $ operatorname{sech}varphi = frac{2e^{-varphi}}{1 + e^{-2varphi}} = 2 sum_{n=0}^{infty} (-1)^n e^{-(2n+1)varphi} $, we obtain



              $$ int_{0}^{infty} frac{varphi}{coshvarphi} , mathrm{d}varphi
              = 2 sum_{n=0}^{infty} (-1)^n int_{0}^{infty} varphi e^{-(2n+1)varphi} , mathrm{d}varphi
              = 2 sum_{n=0}^{infty} frac{(-1)^n}{(2n+1)^2}
              = 2G. $$




            • Taking integration by parts,



              begin{align*}
              int_{0}^{frac{pi}{3}} frac{theta}{costheta} , mathrm{d}theta
              &= left[ - theta log left( tan left( frac{pi}{4} - frac{theta}{2} right) right) right]_{0}^{frac{pi}{3}} + int_{0}^{frac{pi}{3}} log left( tan left( frac{pi}{4} - frac{theta}{2} right) right) , mathrm{d}theta \
              &= frac{pi}{3} log left( 2 + sqrt{3} right) + 2 int_{frac{pi}{12}}^{frac{pi}{4}} log left( tan theta right) , mathrm{d}theta.
              end{align*}



              This can be computed by using the Fourier series $log left( tan theta right) = - 2 sum_{n=0}^{infty} frac{cos(4n+2)theta}{2n+1} $ to yield



              begin{align*}
              int_{0}^{frac{pi}{3}} frac{theta}{costheta} , mathrm{d}theta
              &= frac{pi}{3} log left( 2 + sqrt{3} right) - 2 sum_{n=0}^{infty} frac{sinleft( frac{pi}{2} (2n+1) right) - sinleft( frac{pi}{6} (2n+1) right)}{(2n+1)^2} \
              &= frac{pi}{3} log left( 2 + sqrt{3} right) - frac{2}{3}G.
              end{align*}




            Combining two result, we obtain the desired answer.






            share|cite|improve this answer









            $endgroup$









            • 1




              $begingroup$
              This is beauty !
              $endgroup$
              – Claude Leibovici
              Jan 22 at 16:10














            10












            10








            10





            $begingroup$

            I will begin with the same approach as in @DavidG's answer, but will settle in a different argument. Let



            $$ J(t) = int_{0}^{infty} frac{operatorname{arsinh}(tx)}{1+x^2} , mathrm{d}x. $$



            Our goal is to compute $J(2)$ using Feynman's trick. Differentiating $J(t)$ and substituting $x=sqrt{u^{-2}-1}$, we obtain



            begin{align*}
            J'(t)
            = int_{0}^{infty} frac{x}{(1+x^2)sqrt{1+t^2x^2}} , mathrm{d}x
            = int_{0}^{1} frac{1}{sqrt{1 - (1-t^2)(1-u^2)}} , mathrm{d}u.
            end{align*}



            So it follows that



            begin{align*}
            J(2)
            &= int_{0}^{2} int_{0}^{1} frac{1}{sqrt{1 - (1-t^2)(1-u^2)}} , mathrm{d}umathrm{d}t \
            &= int_{0}^{1} int_{0}^{1} frac{1}{sqrt{1 - (1-t^2)(1-u^2)}} , mathrm{d}umathrm{d}t
            + int_{1}^{2} int_{0}^{1} frac{1}{sqrt{1 + (t^2-1)(1-u^2)}} , mathrm{d}umathrm{d}t.
            end{align*}



            The inner integral is easily computed, yielding



            begin{align*}
            J(2)
            &= int_{0}^{1} frac{operatorname{artanh}left( sqrt{1 - t^2} right)}{sqrt{1 - t^2}} , mathrm{d}t
            + int_{1}^{2} frac{arctanleft(sqrt{t^2-1}right)}{sqrt{t^2 - 1}} , mathrm{d}t.
            end{align*}



            Now we substitute $t = operatorname{sech} varphi$ for the first integral and $t = sec theta$ for the second integral. This yields



            begin{align*}
            J(2)
            &= int_{0}^{infty} frac{varphi}{coshvarphi} , mathrm{d}varphi
            + int_{0}^{frac{pi}{3}} frac{theta}{costheta} , mathrm{d}theta.
            end{align*}



            These integrals can be computed as follows:





            • Using $ operatorname{sech}varphi = frac{2e^{-varphi}}{1 + e^{-2varphi}} = 2 sum_{n=0}^{infty} (-1)^n e^{-(2n+1)varphi} $, we obtain



              $$ int_{0}^{infty} frac{varphi}{coshvarphi} , mathrm{d}varphi
              = 2 sum_{n=0}^{infty} (-1)^n int_{0}^{infty} varphi e^{-(2n+1)varphi} , mathrm{d}varphi
              = 2 sum_{n=0}^{infty} frac{(-1)^n}{(2n+1)^2}
              = 2G. $$




            • Taking integration by parts,



              begin{align*}
              int_{0}^{frac{pi}{3}} frac{theta}{costheta} , mathrm{d}theta
              &= left[ - theta log left( tan left( frac{pi}{4} - frac{theta}{2} right) right) right]_{0}^{frac{pi}{3}} + int_{0}^{frac{pi}{3}} log left( tan left( frac{pi}{4} - frac{theta}{2} right) right) , mathrm{d}theta \
              &= frac{pi}{3} log left( 2 + sqrt{3} right) + 2 int_{frac{pi}{12}}^{frac{pi}{4}} log left( tan theta right) , mathrm{d}theta.
              end{align*}



              This can be computed by using the Fourier series $log left( tan theta right) = - 2 sum_{n=0}^{infty} frac{cos(4n+2)theta}{2n+1} $ to yield



              begin{align*}
              int_{0}^{frac{pi}{3}} frac{theta}{costheta} , mathrm{d}theta
              &= frac{pi}{3} log left( 2 + sqrt{3} right) - 2 sum_{n=0}^{infty} frac{sinleft( frac{pi}{2} (2n+1) right) - sinleft( frac{pi}{6} (2n+1) right)}{(2n+1)^2} \
              &= frac{pi}{3} log left( 2 + sqrt{3} right) - frac{2}{3}G.
              end{align*}




            Combining two result, we obtain the desired answer.






            share|cite|improve this answer









            $endgroup$



            I will begin with the same approach as in @DavidG's answer, but will settle in a different argument. Let



            $$ J(t) = int_{0}^{infty} frac{operatorname{arsinh}(tx)}{1+x^2} , mathrm{d}x. $$



            Our goal is to compute $J(2)$ using Feynman's trick. Differentiating $J(t)$ and substituting $x=sqrt{u^{-2}-1}$, we obtain



            begin{align*}
            J'(t)
            = int_{0}^{infty} frac{x}{(1+x^2)sqrt{1+t^2x^2}} , mathrm{d}x
            = int_{0}^{1} frac{1}{sqrt{1 - (1-t^2)(1-u^2)}} , mathrm{d}u.
            end{align*}



            So it follows that



            begin{align*}
            J(2)
            &= int_{0}^{2} int_{0}^{1} frac{1}{sqrt{1 - (1-t^2)(1-u^2)}} , mathrm{d}umathrm{d}t \
            &= int_{0}^{1} int_{0}^{1} frac{1}{sqrt{1 - (1-t^2)(1-u^2)}} , mathrm{d}umathrm{d}t
            + int_{1}^{2} int_{0}^{1} frac{1}{sqrt{1 + (t^2-1)(1-u^2)}} , mathrm{d}umathrm{d}t.
            end{align*}



            The inner integral is easily computed, yielding



            begin{align*}
            J(2)
            &= int_{0}^{1} frac{operatorname{artanh}left( sqrt{1 - t^2} right)}{sqrt{1 - t^2}} , mathrm{d}t
            + int_{1}^{2} frac{arctanleft(sqrt{t^2-1}right)}{sqrt{t^2 - 1}} , mathrm{d}t.
            end{align*}



            Now we substitute $t = operatorname{sech} varphi$ for the first integral and $t = sec theta$ for the second integral. This yields



            begin{align*}
            J(2)
            &= int_{0}^{infty} frac{varphi}{coshvarphi} , mathrm{d}varphi
            + int_{0}^{frac{pi}{3}} frac{theta}{costheta} , mathrm{d}theta.
            end{align*}



            These integrals can be computed as follows:





            • Using $ operatorname{sech}varphi = frac{2e^{-varphi}}{1 + e^{-2varphi}} = 2 sum_{n=0}^{infty} (-1)^n e^{-(2n+1)varphi} $, we obtain



              $$ int_{0}^{infty} frac{varphi}{coshvarphi} , mathrm{d}varphi
              = 2 sum_{n=0}^{infty} (-1)^n int_{0}^{infty} varphi e^{-(2n+1)varphi} , mathrm{d}varphi
              = 2 sum_{n=0}^{infty} frac{(-1)^n}{(2n+1)^2}
              = 2G. $$




            • Taking integration by parts,



              begin{align*}
              int_{0}^{frac{pi}{3}} frac{theta}{costheta} , mathrm{d}theta
              &= left[ - theta log left( tan left( frac{pi}{4} - frac{theta}{2} right) right) right]_{0}^{frac{pi}{3}} + int_{0}^{frac{pi}{3}} log left( tan left( frac{pi}{4} - frac{theta}{2} right) right) , mathrm{d}theta \
              &= frac{pi}{3} log left( 2 + sqrt{3} right) + 2 int_{frac{pi}{12}}^{frac{pi}{4}} log left( tan theta right) , mathrm{d}theta.
              end{align*}



              This can be computed by using the Fourier series $log left( tan theta right) = - 2 sum_{n=0}^{infty} frac{cos(4n+2)theta}{2n+1} $ to yield



              begin{align*}
              int_{0}^{frac{pi}{3}} frac{theta}{costheta} , mathrm{d}theta
              &= frac{pi}{3} log left( 2 + sqrt{3} right) - 2 sum_{n=0}^{infty} frac{sinleft( frac{pi}{2} (2n+1) right) - sinleft( frac{pi}{6} (2n+1) right)}{(2n+1)^2} \
              &= frac{pi}{3} log left( 2 + sqrt{3} right) - frac{2}{3}G.
              end{align*}




            Combining two result, we obtain the desired answer.







            share|cite|improve this answer












            share|cite|improve this answer



            share|cite|improve this answer










            answered Jan 22 at 11:24









            Sangchul LeeSangchul Lee

            95.5k12171279




            95.5k12171279








            • 1




              $begingroup$
              This is beauty !
              $endgroup$
              – Claude Leibovici
              Jan 22 at 16:10














            • 1




              $begingroup$
              This is beauty !
              $endgroup$
              – Claude Leibovici
              Jan 22 at 16:10








            1




            1




            $begingroup$
            This is beauty !
            $endgroup$
            – Claude Leibovici
            Jan 22 at 16:10




            $begingroup$
            This is beauty !
            $endgroup$
            – Claude Leibovici
            Jan 22 at 16:10











            10












            $begingroup$

            On the path of Kemono Chen...



            begin{align}J&=int_0^{pi/2}operatorname{arcsinh}(2tan x)dxend{align}



            Perform the change of variable $y=operatorname{arcsinh}(2tan x)$,



            begin{align}J&=int_0^{+infty}frac{2xcosh x}{4+sinh^2 x},dx\
            &=int_0^{+infty}frac{4xleft(text{e}^{x}+text{e}^{-x}right)}{14+text{e}^{2x}+text{e}^{-2x}},dx\
            &=int_0^{+infty}frac{4xtext{e}^{-x}left(text{e}^{2x}+1right)}{14+text{e}^{2x}+text{e}^{-2x}},dx\
            end{align}



            Perform the change of variable $y=text{e}^{-x}$,



            begin{align}J&=-int_0^1 frac{4ln xleft(1+frac{1}{x^2}right)}{14+x^2+frac{1}{x^2}}\
            &=-int_0^1 frac{4ln xleft(1+x^2right)}{x^4+14x^2+1}\
            &=left[-arctanleft(frac{4x}{1-x^2}right)ln xright]_0^1+int_0^1 frac{arctanleft(frac{4x}{1-x^2}right)}{x},dx\
            &=int_0^1 frac{arctanleft(frac{4x}{1-x^2}right)}{x},dx\
            &=int_0^1 frac{arctanleft(left(2+sqrt{3}right)xright)}{x},dx+int_0^1 frac{arctanleft(left(2-sqrt{3}right)xright)}{x},dx\
            end{align}



            In the first integral perform the change of variable $y=left(2+sqrt{3}right)x$,



            In the second integral perform the change of variable $y=left(2-sqrt{3}right)x$,



            begin{align}J&=int_0^{2+sqrt{3}}frac{arctan x}{x},dx+int_0^{2-sqrt{3}}frac{arctan x}{x},dx\
            &=Big[arctan xln xBig]_0^{2+sqrt{3}}-int_0^{2+sqrt{3}}frac{ln x}{1+x^2},dx+Big[arctan xln xBig]_0^{2-sqrt{3}}-int_0^{2-sqrt{3}}frac{ln x}{1+x^2},dx\
            &=frac{5pi}{12}lnleft(2+sqrt{3}right)-int_0^{2+sqrt{3}}frac{ln x}{1+x^2},dx+frac{pi}{12}lnleft(2-sqrt{3}right)-int_0^{2-sqrt{3}}frac{ln x}{1+x^2},dx\
            &=frac{pi}{3}lnleft(2+sqrt{3}right)-int_0^{2+sqrt{3}}frac{ln x}{1+x^2},dx-int_0^{2-sqrt{3}}frac{ln x}{1+x^2},dx
            end{align}



            In the first integral perform the change of variable $y=dfrac{1}{x}$,



            begin{align}J&=frac{pi}{3}lnleft(2+sqrt{3}right)+int_{2-sqrt{3}}^{+infty}frac{ln x}{1+x^2},dx-int_0^{2-sqrt{3}}frac{ln x}{1+x^2},dx\
            &=frac{pi}{3}lnleft(2+sqrt{3}right)+int_0^{+infty}frac{ln x}{1+x^2},dx-2int_0^{2-sqrt{3}}frac{ln x}{1+x^2},dx\
            &=frac{pi}{3}lnleft(2+sqrt{3}right)-2int_0^{2-sqrt{3}}frac{ln x}{1+x^2},dx\
            end{align}



            Perform the change of variable $y=tan x$,



            begin{align}J&=frac{pi}{3}lnleft(2+sqrt{3}right)-2int_0^{frac{pi}{12}}lnleft(tan xright),dx\
            end{align}



            It is well known that,



            begin{align}
            int_0^{frac{pi}{12}}lnleft(tan xright),dx=-frac{2}{3}text{G}
            end{align}



            (see: Integral: $int_0^{pi/12} ln(tan x),dx$ )



            Thus,



            begin{align}J&=frac{pi}{3}lnleft(2+sqrt{3}right)-2times -frac{2}{3}text{G}\
            &=boxed{frac{pi}{3}lnleft(2+sqrt{3}right)+frac{4}{3}text{G}}
            end{align}



            NB:



            Observe that,



            begin{align}2-sqrt{3}&=frac{1}{2+sqrt{3}}\
            lnleft(2-sqrt{3}right)&=-lnleft(2+sqrt{3}right)\
            int_0^infty frac{ln x}{1+x^2},dx&=0
            end{align}

            (perform the change of variable $y=dfrac{1}{x}$ )






            share|cite|improve this answer











            $endgroup$













            • $begingroup$
              This is beauty !
              $endgroup$
              – Claude Leibovici
              Jan 22 at 16:10










            • $begingroup$
              How did you do $$intarctanfrac{4x}{1-x^2}frac{mathrm dx}{x}=intarctan(2+sqrt3)xfrac{mathrm dx}{x}+intarctan(2-sqrt3)xfrac{mathrm dx}{x}$$
              $endgroup$
              – clathratus
              Jan 22 at 16:10








            • 1




              $begingroup$
              @Clathratus: if $ab<1$then begin{align}arctan a+arctan b=arctanleft(frac{a+b}{1-ab}right)end{align}
              $endgroup$
              – FDP
              Jan 22 at 16:23
















            10












            $begingroup$

            On the path of Kemono Chen...



            begin{align}J&=int_0^{pi/2}operatorname{arcsinh}(2tan x)dxend{align}



            Perform the change of variable $y=operatorname{arcsinh}(2tan x)$,



            begin{align}J&=int_0^{+infty}frac{2xcosh x}{4+sinh^2 x},dx\
            &=int_0^{+infty}frac{4xleft(text{e}^{x}+text{e}^{-x}right)}{14+text{e}^{2x}+text{e}^{-2x}},dx\
            &=int_0^{+infty}frac{4xtext{e}^{-x}left(text{e}^{2x}+1right)}{14+text{e}^{2x}+text{e}^{-2x}},dx\
            end{align}



            Perform the change of variable $y=text{e}^{-x}$,



            begin{align}J&=-int_0^1 frac{4ln xleft(1+frac{1}{x^2}right)}{14+x^2+frac{1}{x^2}}\
            &=-int_0^1 frac{4ln xleft(1+x^2right)}{x^4+14x^2+1}\
            &=left[-arctanleft(frac{4x}{1-x^2}right)ln xright]_0^1+int_0^1 frac{arctanleft(frac{4x}{1-x^2}right)}{x},dx\
            &=int_0^1 frac{arctanleft(frac{4x}{1-x^2}right)}{x},dx\
            &=int_0^1 frac{arctanleft(left(2+sqrt{3}right)xright)}{x},dx+int_0^1 frac{arctanleft(left(2-sqrt{3}right)xright)}{x},dx\
            end{align}



            In the first integral perform the change of variable $y=left(2+sqrt{3}right)x$,



            In the second integral perform the change of variable $y=left(2-sqrt{3}right)x$,



            begin{align}J&=int_0^{2+sqrt{3}}frac{arctan x}{x},dx+int_0^{2-sqrt{3}}frac{arctan x}{x},dx\
            &=Big[arctan xln xBig]_0^{2+sqrt{3}}-int_0^{2+sqrt{3}}frac{ln x}{1+x^2},dx+Big[arctan xln xBig]_0^{2-sqrt{3}}-int_0^{2-sqrt{3}}frac{ln x}{1+x^2},dx\
            &=frac{5pi}{12}lnleft(2+sqrt{3}right)-int_0^{2+sqrt{3}}frac{ln x}{1+x^2},dx+frac{pi}{12}lnleft(2-sqrt{3}right)-int_0^{2-sqrt{3}}frac{ln x}{1+x^2},dx\
            &=frac{pi}{3}lnleft(2+sqrt{3}right)-int_0^{2+sqrt{3}}frac{ln x}{1+x^2},dx-int_0^{2-sqrt{3}}frac{ln x}{1+x^2},dx
            end{align}



            In the first integral perform the change of variable $y=dfrac{1}{x}$,



            begin{align}J&=frac{pi}{3}lnleft(2+sqrt{3}right)+int_{2-sqrt{3}}^{+infty}frac{ln x}{1+x^2},dx-int_0^{2-sqrt{3}}frac{ln x}{1+x^2},dx\
            &=frac{pi}{3}lnleft(2+sqrt{3}right)+int_0^{+infty}frac{ln x}{1+x^2},dx-2int_0^{2-sqrt{3}}frac{ln x}{1+x^2},dx\
            &=frac{pi}{3}lnleft(2+sqrt{3}right)-2int_0^{2-sqrt{3}}frac{ln x}{1+x^2},dx\
            end{align}



            Perform the change of variable $y=tan x$,



            begin{align}J&=frac{pi}{3}lnleft(2+sqrt{3}right)-2int_0^{frac{pi}{12}}lnleft(tan xright),dx\
            end{align}



            It is well known that,



            begin{align}
            int_0^{frac{pi}{12}}lnleft(tan xright),dx=-frac{2}{3}text{G}
            end{align}



            (see: Integral: $int_0^{pi/12} ln(tan x),dx$ )



            Thus,



            begin{align}J&=frac{pi}{3}lnleft(2+sqrt{3}right)-2times -frac{2}{3}text{G}\
            &=boxed{frac{pi}{3}lnleft(2+sqrt{3}right)+frac{4}{3}text{G}}
            end{align}



            NB:



            Observe that,



            begin{align}2-sqrt{3}&=frac{1}{2+sqrt{3}}\
            lnleft(2-sqrt{3}right)&=-lnleft(2+sqrt{3}right)\
            int_0^infty frac{ln x}{1+x^2},dx&=0
            end{align}

            (perform the change of variable $y=dfrac{1}{x}$ )






            share|cite|improve this answer











            $endgroup$













            • $begingroup$
              This is beauty !
              $endgroup$
              – Claude Leibovici
              Jan 22 at 16:10










            • $begingroup$
              How did you do $$intarctanfrac{4x}{1-x^2}frac{mathrm dx}{x}=intarctan(2+sqrt3)xfrac{mathrm dx}{x}+intarctan(2-sqrt3)xfrac{mathrm dx}{x}$$
              $endgroup$
              – clathratus
              Jan 22 at 16:10








            • 1




              $begingroup$
              @Clathratus: if $ab<1$then begin{align}arctan a+arctan b=arctanleft(frac{a+b}{1-ab}right)end{align}
              $endgroup$
              – FDP
              Jan 22 at 16:23














            10












            10








            10





            $begingroup$

            On the path of Kemono Chen...



            begin{align}J&=int_0^{pi/2}operatorname{arcsinh}(2tan x)dxend{align}



            Perform the change of variable $y=operatorname{arcsinh}(2tan x)$,



            begin{align}J&=int_0^{+infty}frac{2xcosh x}{4+sinh^2 x},dx\
            &=int_0^{+infty}frac{4xleft(text{e}^{x}+text{e}^{-x}right)}{14+text{e}^{2x}+text{e}^{-2x}},dx\
            &=int_0^{+infty}frac{4xtext{e}^{-x}left(text{e}^{2x}+1right)}{14+text{e}^{2x}+text{e}^{-2x}},dx\
            end{align}



            Perform the change of variable $y=text{e}^{-x}$,



            begin{align}J&=-int_0^1 frac{4ln xleft(1+frac{1}{x^2}right)}{14+x^2+frac{1}{x^2}}\
            &=-int_0^1 frac{4ln xleft(1+x^2right)}{x^4+14x^2+1}\
            &=left[-arctanleft(frac{4x}{1-x^2}right)ln xright]_0^1+int_0^1 frac{arctanleft(frac{4x}{1-x^2}right)}{x},dx\
            &=int_0^1 frac{arctanleft(frac{4x}{1-x^2}right)}{x},dx\
            &=int_0^1 frac{arctanleft(left(2+sqrt{3}right)xright)}{x},dx+int_0^1 frac{arctanleft(left(2-sqrt{3}right)xright)}{x},dx\
            end{align}



            In the first integral perform the change of variable $y=left(2+sqrt{3}right)x$,



            In the second integral perform the change of variable $y=left(2-sqrt{3}right)x$,



            begin{align}J&=int_0^{2+sqrt{3}}frac{arctan x}{x},dx+int_0^{2-sqrt{3}}frac{arctan x}{x},dx\
            &=Big[arctan xln xBig]_0^{2+sqrt{3}}-int_0^{2+sqrt{3}}frac{ln x}{1+x^2},dx+Big[arctan xln xBig]_0^{2-sqrt{3}}-int_0^{2-sqrt{3}}frac{ln x}{1+x^2},dx\
            &=frac{5pi}{12}lnleft(2+sqrt{3}right)-int_0^{2+sqrt{3}}frac{ln x}{1+x^2},dx+frac{pi}{12}lnleft(2-sqrt{3}right)-int_0^{2-sqrt{3}}frac{ln x}{1+x^2},dx\
            &=frac{pi}{3}lnleft(2+sqrt{3}right)-int_0^{2+sqrt{3}}frac{ln x}{1+x^2},dx-int_0^{2-sqrt{3}}frac{ln x}{1+x^2},dx
            end{align}



            In the first integral perform the change of variable $y=dfrac{1}{x}$,



            begin{align}J&=frac{pi}{3}lnleft(2+sqrt{3}right)+int_{2-sqrt{3}}^{+infty}frac{ln x}{1+x^2},dx-int_0^{2-sqrt{3}}frac{ln x}{1+x^2},dx\
            &=frac{pi}{3}lnleft(2+sqrt{3}right)+int_0^{+infty}frac{ln x}{1+x^2},dx-2int_0^{2-sqrt{3}}frac{ln x}{1+x^2},dx\
            &=frac{pi}{3}lnleft(2+sqrt{3}right)-2int_0^{2-sqrt{3}}frac{ln x}{1+x^2},dx\
            end{align}



            Perform the change of variable $y=tan x$,



            begin{align}J&=frac{pi}{3}lnleft(2+sqrt{3}right)-2int_0^{frac{pi}{12}}lnleft(tan xright),dx\
            end{align}



            It is well known that,



            begin{align}
            int_0^{frac{pi}{12}}lnleft(tan xright),dx=-frac{2}{3}text{G}
            end{align}



            (see: Integral: $int_0^{pi/12} ln(tan x),dx$ )



            Thus,



            begin{align}J&=frac{pi}{3}lnleft(2+sqrt{3}right)-2times -frac{2}{3}text{G}\
            &=boxed{frac{pi}{3}lnleft(2+sqrt{3}right)+frac{4}{3}text{G}}
            end{align}



            NB:



            Observe that,



            begin{align}2-sqrt{3}&=frac{1}{2+sqrt{3}}\
            lnleft(2-sqrt{3}right)&=-lnleft(2+sqrt{3}right)\
            int_0^infty frac{ln x}{1+x^2},dx&=0
            end{align}

            (perform the change of variable $y=dfrac{1}{x}$ )






            share|cite|improve this answer











            $endgroup$



            On the path of Kemono Chen...



            begin{align}J&=int_0^{pi/2}operatorname{arcsinh}(2tan x)dxend{align}



            Perform the change of variable $y=operatorname{arcsinh}(2tan x)$,



            begin{align}J&=int_0^{+infty}frac{2xcosh x}{4+sinh^2 x},dx\
            &=int_0^{+infty}frac{4xleft(text{e}^{x}+text{e}^{-x}right)}{14+text{e}^{2x}+text{e}^{-2x}},dx\
            &=int_0^{+infty}frac{4xtext{e}^{-x}left(text{e}^{2x}+1right)}{14+text{e}^{2x}+text{e}^{-2x}},dx\
            end{align}



            Perform the change of variable $y=text{e}^{-x}$,



            begin{align}J&=-int_0^1 frac{4ln xleft(1+frac{1}{x^2}right)}{14+x^2+frac{1}{x^2}}\
            &=-int_0^1 frac{4ln xleft(1+x^2right)}{x^4+14x^2+1}\
            &=left[-arctanleft(frac{4x}{1-x^2}right)ln xright]_0^1+int_0^1 frac{arctanleft(frac{4x}{1-x^2}right)}{x},dx\
            &=int_0^1 frac{arctanleft(frac{4x}{1-x^2}right)}{x},dx\
            &=int_0^1 frac{arctanleft(left(2+sqrt{3}right)xright)}{x},dx+int_0^1 frac{arctanleft(left(2-sqrt{3}right)xright)}{x},dx\
            end{align}



            In the first integral perform the change of variable $y=left(2+sqrt{3}right)x$,



            In the second integral perform the change of variable $y=left(2-sqrt{3}right)x$,



            begin{align}J&=int_0^{2+sqrt{3}}frac{arctan x}{x},dx+int_0^{2-sqrt{3}}frac{arctan x}{x},dx\
            &=Big[arctan xln xBig]_0^{2+sqrt{3}}-int_0^{2+sqrt{3}}frac{ln x}{1+x^2},dx+Big[arctan xln xBig]_0^{2-sqrt{3}}-int_0^{2-sqrt{3}}frac{ln x}{1+x^2},dx\
            &=frac{5pi}{12}lnleft(2+sqrt{3}right)-int_0^{2+sqrt{3}}frac{ln x}{1+x^2},dx+frac{pi}{12}lnleft(2-sqrt{3}right)-int_0^{2-sqrt{3}}frac{ln x}{1+x^2},dx\
            &=frac{pi}{3}lnleft(2+sqrt{3}right)-int_0^{2+sqrt{3}}frac{ln x}{1+x^2},dx-int_0^{2-sqrt{3}}frac{ln x}{1+x^2},dx
            end{align}



            In the first integral perform the change of variable $y=dfrac{1}{x}$,



            begin{align}J&=frac{pi}{3}lnleft(2+sqrt{3}right)+int_{2-sqrt{3}}^{+infty}frac{ln x}{1+x^2},dx-int_0^{2-sqrt{3}}frac{ln x}{1+x^2},dx\
            &=frac{pi}{3}lnleft(2+sqrt{3}right)+int_0^{+infty}frac{ln x}{1+x^2},dx-2int_0^{2-sqrt{3}}frac{ln x}{1+x^2},dx\
            &=frac{pi}{3}lnleft(2+sqrt{3}right)-2int_0^{2-sqrt{3}}frac{ln x}{1+x^2},dx\
            end{align}



            Perform the change of variable $y=tan x$,



            begin{align}J&=frac{pi}{3}lnleft(2+sqrt{3}right)-2int_0^{frac{pi}{12}}lnleft(tan xright),dx\
            end{align}



            It is well known that,



            begin{align}
            int_0^{frac{pi}{12}}lnleft(tan xright),dx=-frac{2}{3}text{G}
            end{align}



            (see: Integral: $int_0^{pi/12} ln(tan x),dx$ )



            Thus,



            begin{align}J&=frac{pi}{3}lnleft(2+sqrt{3}right)-2times -frac{2}{3}text{G}\
            &=boxed{frac{pi}{3}lnleft(2+sqrt{3}right)+frac{4}{3}text{G}}
            end{align}



            NB:



            Observe that,



            begin{align}2-sqrt{3}&=frac{1}{2+sqrt{3}}\
            lnleft(2-sqrt{3}right)&=-lnleft(2+sqrt{3}right)\
            int_0^infty frac{ln x}{1+x^2},dx&=0
            end{align}

            (perform the change of variable $y=dfrac{1}{x}$ )







            share|cite|improve this answer














            share|cite|improve this answer



            share|cite|improve this answer








            edited Jan 22 at 13:40

























            answered Jan 22 at 13:35









            FDPFDP

            6,02211829




            6,02211829












            • $begingroup$
              This is beauty !
              $endgroup$
              – Claude Leibovici
              Jan 22 at 16:10










            • $begingroup$
              How did you do $$intarctanfrac{4x}{1-x^2}frac{mathrm dx}{x}=intarctan(2+sqrt3)xfrac{mathrm dx}{x}+intarctan(2-sqrt3)xfrac{mathrm dx}{x}$$
              $endgroup$
              – clathratus
              Jan 22 at 16:10








            • 1




              $begingroup$
              @Clathratus: if $ab<1$then begin{align}arctan a+arctan b=arctanleft(frac{a+b}{1-ab}right)end{align}
              $endgroup$
              – FDP
              Jan 22 at 16:23


















            • $begingroup$
              This is beauty !
              $endgroup$
              – Claude Leibovici
              Jan 22 at 16:10










            • $begingroup$
              How did you do $$intarctanfrac{4x}{1-x^2}frac{mathrm dx}{x}=intarctan(2+sqrt3)xfrac{mathrm dx}{x}+intarctan(2-sqrt3)xfrac{mathrm dx}{x}$$
              $endgroup$
              – clathratus
              Jan 22 at 16:10








            • 1




              $begingroup$
              @Clathratus: if $ab<1$then begin{align}arctan a+arctan b=arctanleft(frac{a+b}{1-ab}right)end{align}
              $endgroup$
              – FDP
              Jan 22 at 16:23
















            $begingroup$
            This is beauty !
            $endgroup$
            – Claude Leibovici
            Jan 22 at 16:10




            $begingroup$
            This is beauty !
            $endgroup$
            – Claude Leibovici
            Jan 22 at 16:10












            $begingroup$
            How did you do $$intarctanfrac{4x}{1-x^2}frac{mathrm dx}{x}=intarctan(2+sqrt3)xfrac{mathrm dx}{x}+intarctan(2-sqrt3)xfrac{mathrm dx}{x}$$
            $endgroup$
            – clathratus
            Jan 22 at 16:10






            $begingroup$
            How did you do $$intarctanfrac{4x}{1-x^2}frac{mathrm dx}{x}=intarctan(2+sqrt3)xfrac{mathrm dx}{x}+intarctan(2-sqrt3)xfrac{mathrm dx}{x}$$
            $endgroup$
            – clathratus
            Jan 22 at 16:10






            1




            1




            $begingroup$
            @Clathratus: if $ab<1$then begin{align}arctan a+arctan b=arctanleft(frac{a+b}{1-ab}right)end{align}
            $endgroup$
            – FDP
            Jan 22 at 16:23




            $begingroup$
            @Clathratus: if $ab<1$then begin{align}arctan a+arctan b=arctanleft(frac{a+b}{1-ab}right)end{align}
            $endgroup$
            – FDP
            Jan 22 at 16:23











            4












            $begingroup$

            Here is another variation along a theme which like @DavidG approach uses Feynman's trick of differentiating under the integral sign.



            Let
            $$I(a) = int_0^{frac{pi}{2}} operatorname{arcsinh} (a tan x) , dx, qquad a > 1.$$
            We are required to find $I(2)$. We start by finding $I(1)$ first, which will be needed later on.



            For $a = 1$ we have
            begin{align}
            I(1) &= int_0^{frac{pi}{2}} operatorname{arcsinh} (tan x) , dx\
            &= int_0^{frac{pi}{2}} ln left (frac{1 + sin x}{cos x} right ) , dx\
            &= int_0^{frac{pi}{2}} ln (1 + sin x) , dx - int_0^{frac{pi}{2}} ln (cos x) , dx.
            end{align}

            Now the first of these integrals can be found, for example, by rewriting it as
            begin{align}
            int_0^{frac{pi}{2}} ln (1 + sin x) , dx &= int_0^{frac{pi}{2}} ln (1 + cos x) , dx\
            &= int_0^{frac{pi}{2}} ln left (frac{1}{2} cos^2 frac{x}{2} right ) , dx\
            &= frac{pi}{2} ln 2 + 4 int_0^{frac{pi}{4}} ln (cos x) , dx
            end{align}

            and then employing the Fourier series representation for $ln (cos x)$ found here. The final result is
            $$int_0^{frac{pi}{2}} ln (1 + sin x) , dx = 2 mathbf{G} - frac{pi}{2} ln 2.$$
            Here $mathbf{G}$ is Catalan's constant.



            The second of the integrals is very well known. Here
            $$int_0^{frac{pi}{2}} ln (cos x) , dx = - frac{pi}{2} ln 2.$$
            Thus $I(1) = 2 mathbf{G}$.



            Moving to the main event, by differentiating under the integral sign with respect to $a$ we have
            begin{align}
            I'(a) &= int_0^{frac{pi}{2}} frac{tan x}{sqrt{a^2 tan^2 x + 1}} , dx\
            &= int_0^{frac{pi}{2}} frac{sin x}{sqrt{a^2 - (a^2 - 1) cos^2 x}} , dx.
            end{align}

            Observe the term $(a^2 - 1)$ is positive since $a > 1$. Letting $u = cos x$ one has
            $$I'(a) = int_0^1 frac{du}{sqrt{a^2 - (a^2 - 1) u^2}} = frac{1}{sqrt{a^2 - 1}} sin^{-1} left (frac{sqrt{a^2 - 1}}{a} right ).$$



            As we require $I(2)$ observe that
            $$I(2) - I(1) = int_1^2 I'(a) , da.$$
            Thus
            $$I(2) = 2 mathbf{G} + int_1^2 frac{1}{sqrt{a^2 - 1}} sin^{-1} left (frac{sqrt{a^2 - 1}}{a} right ) , da.$$
            Integrating by parts leads to
            $$I(2) = 2 mathbf{G} + frac{pi}{3} ln (2 + sqrt{3}) - int_1^2 frac{cosh^{-1} a}{a sqrt{a^2 - 1}} , da.$$
            Now let $a = cosh t$. This gives
            $$I(2) = 2mathbf{G} + frac{pi}{3} ln (2 + sqrt{3}) - int_0^{ln(2 + sqrt{3})} frac{t}{cosh t} , dt.$$
            Then let $t = ln y$. This gives
            $$I(2) = 2mathbf{G} + frac{pi}{3} ln (2 + sqrt{3}) - 2int_1^{2 + sqrt{3}} frac{ln y}{1 + y^2} , dy.$$
            Now let $y = tan theta$. Then we have
            $$I(2) = 2mathbf{G} + frac{pi}{3} ln (2 + sqrt{3}) - 2int_{frac{pi}{4}}^{frac{5pi}{12}} ln (tan theta) , dtheta.$$
            Finally, enforcing a substitution of $theta mapsto dfrac{pi}{2} - theta$ leads to
            begin{align}
            I(2) &= 2mathbf{G} + frac{pi}{3} ln (2 + sqrt{3}) + 2 int_{frac{pi}{12}}^{frac{pi}{4}} ln (tan theta) , dtheta\
            &= 2mathbf{G} + frac{pi}{3} ln (2 + sqrt{3}) + 2int_0^{frac{pi}{4}} ln (tan theta) , dtheta - 2 int_0^{frac{pi}{12}} ln (tan theta) , dtheta.
            end{align}

            For the first of the integrals we have
            $$int_0^{frac{pi}{4}} ln (tan x) , dx = -mathbf{G}.$$
            While for the second of the integrals we have
            $$int_0^{frac{pi}{12}} ln (tan x) , dx = -frac{2}{3} mathbf{G}.$$
            So finally
            $$I(2) = 2mathbf{G} + frac{pi}{3} ln (2 + sqrt{3}) - 2mathbf{G} + frac{4}{3} mathbf{G},$$
            or
            $$int_0^{frac{pi}{2}} operatorname{arcsinh} (2 tan x) , dx = frac{pi}{3} ln (2 + sqrt{3}) + frac{4}{3} mathbf{G},$$
            as announced.






            share|cite|improve this answer









            $endgroup$









            • 1




              $begingroup$
              begin{align}A&=int_0^{frac{pi}{4}}ln(cos x),dx\ B&=int_0^{frac{pi}{4}}ln(sin x),dx\ B-A&=int_0^{frac{pi}{4}}ln(tan x),dx\ &=-text{G}\ A+B&=int_0^{frac{pi}{4}}ln(sin xcos x),dx\ &=int_0^{frac{pi}{4}}lnleft(frac{sin(2x)}{2}right),dx\ &=frac{1}{2}int_0^{frac{pi}{2}}lnleft(frac{sin x}{2}right),dx\ &=frac{1}{2}int_0^{frac{pi}{2}}lnleft(sin xright),dx-frac{pi}{4}ln 2\ &=-frac{1}{2}piln 2\ A&=frac{1}{2}left(A+B-left(B-Aright)right)\ &=frac{1}{2}text{G}-frac{1}{4}piln 2 end{align}
              $endgroup$
              – FDP
              Jan 23 at 13:44










            • $begingroup$
              Yes, very nice.
              $endgroup$
              – omegadot
              Jan 24 at 1:29
















            4












            $begingroup$

            Here is another variation along a theme which like @DavidG approach uses Feynman's trick of differentiating under the integral sign.



            Let
            $$I(a) = int_0^{frac{pi}{2}} operatorname{arcsinh} (a tan x) , dx, qquad a > 1.$$
            We are required to find $I(2)$. We start by finding $I(1)$ first, which will be needed later on.



            For $a = 1$ we have
            begin{align}
            I(1) &= int_0^{frac{pi}{2}} operatorname{arcsinh} (tan x) , dx\
            &= int_0^{frac{pi}{2}} ln left (frac{1 + sin x}{cos x} right ) , dx\
            &= int_0^{frac{pi}{2}} ln (1 + sin x) , dx - int_0^{frac{pi}{2}} ln (cos x) , dx.
            end{align}

            Now the first of these integrals can be found, for example, by rewriting it as
            begin{align}
            int_0^{frac{pi}{2}} ln (1 + sin x) , dx &= int_0^{frac{pi}{2}} ln (1 + cos x) , dx\
            &= int_0^{frac{pi}{2}} ln left (frac{1}{2} cos^2 frac{x}{2} right ) , dx\
            &= frac{pi}{2} ln 2 + 4 int_0^{frac{pi}{4}} ln (cos x) , dx
            end{align}

            and then employing the Fourier series representation for $ln (cos x)$ found here. The final result is
            $$int_0^{frac{pi}{2}} ln (1 + sin x) , dx = 2 mathbf{G} - frac{pi}{2} ln 2.$$
            Here $mathbf{G}$ is Catalan's constant.



            The second of the integrals is very well known. Here
            $$int_0^{frac{pi}{2}} ln (cos x) , dx = - frac{pi}{2} ln 2.$$
            Thus $I(1) = 2 mathbf{G}$.



            Moving to the main event, by differentiating under the integral sign with respect to $a$ we have
            begin{align}
            I'(a) &= int_0^{frac{pi}{2}} frac{tan x}{sqrt{a^2 tan^2 x + 1}} , dx\
            &= int_0^{frac{pi}{2}} frac{sin x}{sqrt{a^2 - (a^2 - 1) cos^2 x}} , dx.
            end{align}

            Observe the term $(a^2 - 1)$ is positive since $a > 1$. Letting $u = cos x$ one has
            $$I'(a) = int_0^1 frac{du}{sqrt{a^2 - (a^2 - 1) u^2}} = frac{1}{sqrt{a^2 - 1}} sin^{-1} left (frac{sqrt{a^2 - 1}}{a} right ).$$



            As we require $I(2)$ observe that
            $$I(2) - I(1) = int_1^2 I'(a) , da.$$
            Thus
            $$I(2) = 2 mathbf{G} + int_1^2 frac{1}{sqrt{a^2 - 1}} sin^{-1} left (frac{sqrt{a^2 - 1}}{a} right ) , da.$$
            Integrating by parts leads to
            $$I(2) = 2 mathbf{G} + frac{pi}{3} ln (2 + sqrt{3}) - int_1^2 frac{cosh^{-1} a}{a sqrt{a^2 - 1}} , da.$$
            Now let $a = cosh t$. This gives
            $$I(2) = 2mathbf{G} + frac{pi}{3} ln (2 + sqrt{3}) - int_0^{ln(2 + sqrt{3})} frac{t}{cosh t} , dt.$$
            Then let $t = ln y$. This gives
            $$I(2) = 2mathbf{G} + frac{pi}{3} ln (2 + sqrt{3}) - 2int_1^{2 + sqrt{3}} frac{ln y}{1 + y^2} , dy.$$
            Now let $y = tan theta$. Then we have
            $$I(2) = 2mathbf{G} + frac{pi}{3} ln (2 + sqrt{3}) - 2int_{frac{pi}{4}}^{frac{5pi}{12}} ln (tan theta) , dtheta.$$
            Finally, enforcing a substitution of $theta mapsto dfrac{pi}{2} - theta$ leads to
            begin{align}
            I(2) &= 2mathbf{G} + frac{pi}{3} ln (2 + sqrt{3}) + 2 int_{frac{pi}{12}}^{frac{pi}{4}} ln (tan theta) , dtheta\
            &= 2mathbf{G} + frac{pi}{3} ln (2 + sqrt{3}) + 2int_0^{frac{pi}{4}} ln (tan theta) , dtheta - 2 int_0^{frac{pi}{12}} ln (tan theta) , dtheta.
            end{align}

            For the first of the integrals we have
            $$int_0^{frac{pi}{4}} ln (tan x) , dx = -mathbf{G}.$$
            While for the second of the integrals we have
            $$int_0^{frac{pi}{12}} ln (tan x) , dx = -frac{2}{3} mathbf{G}.$$
            So finally
            $$I(2) = 2mathbf{G} + frac{pi}{3} ln (2 + sqrt{3}) - 2mathbf{G} + frac{4}{3} mathbf{G},$$
            or
            $$int_0^{frac{pi}{2}} operatorname{arcsinh} (2 tan x) , dx = frac{pi}{3} ln (2 + sqrt{3}) + frac{4}{3} mathbf{G},$$
            as announced.






            share|cite|improve this answer









            $endgroup$









            • 1




              $begingroup$
              begin{align}A&=int_0^{frac{pi}{4}}ln(cos x),dx\ B&=int_0^{frac{pi}{4}}ln(sin x),dx\ B-A&=int_0^{frac{pi}{4}}ln(tan x),dx\ &=-text{G}\ A+B&=int_0^{frac{pi}{4}}ln(sin xcos x),dx\ &=int_0^{frac{pi}{4}}lnleft(frac{sin(2x)}{2}right),dx\ &=frac{1}{2}int_0^{frac{pi}{2}}lnleft(frac{sin x}{2}right),dx\ &=frac{1}{2}int_0^{frac{pi}{2}}lnleft(sin xright),dx-frac{pi}{4}ln 2\ &=-frac{1}{2}piln 2\ A&=frac{1}{2}left(A+B-left(B-Aright)right)\ &=frac{1}{2}text{G}-frac{1}{4}piln 2 end{align}
              $endgroup$
              – FDP
              Jan 23 at 13:44










            • $begingroup$
              Yes, very nice.
              $endgroup$
              – omegadot
              Jan 24 at 1:29














            4












            4








            4





            $begingroup$

            Here is another variation along a theme which like @DavidG approach uses Feynman's trick of differentiating under the integral sign.



            Let
            $$I(a) = int_0^{frac{pi}{2}} operatorname{arcsinh} (a tan x) , dx, qquad a > 1.$$
            We are required to find $I(2)$. We start by finding $I(1)$ first, which will be needed later on.



            For $a = 1$ we have
            begin{align}
            I(1) &= int_0^{frac{pi}{2}} operatorname{arcsinh} (tan x) , dx\
            &= int_0^{frac{pi}{2}} ln left (frac{1 + sin x}{cos x} right ) , dx\
            &= int_0^{frac{pi}{2}} ln (1 + sin x) , dx - int_0^{frac{pi}{2}} ln (cos x) , dx.
            end{align}

            Now the first of these integrals can be found, for example, by rewriting it as
            begin{align}
            int_0^{frac{pi}{2}} ln (1 + sin x) , dx &= int_0^{frac{pi}{2}} ln (1 + cos x) , dx\
            &= int_0^{frac{pi}{2}} ln left (frac{1}{2} cos^2 frac{x}{2} right ) , dx\
            &= frac{pi}{2} ln 2 + 4 int_0^{frac{pi}{4}} ln (cos x) , dx
            end{align}

            and then employing the Fourier series representation for $ln (cos x)$ found here. The final result is
            $$int_0^{frac{pi}{2}} ln (1 + sin x) , dx = 2 mathbf{G} - frac{pi}{2} ln 2.$$
            Here $mathbf{G}$ is Catalan's constant.



            The second of the integrals is very well known. Here
            $$int_0^{frac{pi}{2}} ln (cos x) , dx = - frac{pi}{2} ln 2.$$
            Thus $I(1) = 2 mathbf{G}$.



            Moving to the main event, by differentiating under the integral sign with respect to $a$ we have
            begin{align}
            I'(a) &= int_0^{frac{pi}{2}} frac{tan x}{sqrt{a^2 tan^2 x + 1}} , dx\
            &= int_0^{frac{pi}{2}} frac{sin x}{sqrt{a^2 - (a^2 - 1) cos^2 x}} , dx.
            end{align}

            Observe the term $(a^2 - 1)$ is positive since $a > 1$. Letting $u = cos x$ one has
            $$I'(a) = int_0^1 frac{du}{sqrt{a^2 - (a^2 - 1) u^2}} = frac{1}{sqrt{a^2 - 1}} sin^{-1} left (frac{sqrt{a^2 - 1}}{a} right ).$$



            As we require $I(2)$ observe that
            $$I(2) - I(1) = int_1^2 I'(a) , da.$$
            Thus
            $$I(2) = 2 mathbf{G} + int_1^2 frac{1}{sqrt{a^2 - 1}} sin^{-1} left (frac{sqrt{a^2 - 1}}{a} right ) , da.$$
            Integrating by parts leads to
            $$I(2) = 2 mathbf{G} + frac{pi}{3} ln (2 + sqrt{3}) - int_1^2 frac{cosh^{-1} a}{a sqrt{a^2 - 1}} , da.$$
            Now let $a = cosh t$. This gives
            $$I(2) = 2mathbf{G} + frac{pi}{3} ln (2 + sqrt{3}) - int_0^{ln(2 + sqrt{3})} frac{t}{cosh t} , dt.$$
            Then let $t = ln y$. This gives
            $$I(2) = 2mathbf{G} + frac{pi}{3} ln (2 + sqrt{3}) - 2int_1^{2 + sqrt{3}} frac{ln y}{1 + y^2} , dy.$$
            Now let $y = tan theta$. Then we have
            $$I(2) = 2mathbf{G} + frac{pi}{3} ln (2 + sqrt{3}) - 2int_{frac{pi}{4}}^{frac{5pi}{12}} ln (tan theta) , dtheta.$$
            Finally, enforcing a substitution of $theta mapsto dfrac{pi}{2} - theta$ leads to
            begin{align}
            I(2) &= 2mathbf{G} + frac{pi}{3} ln (2 + sqrt{3}) + 2 int_{frac{pi}{12}}^{frac{pi}{4}} ln (tan theta) , dtheta\
            &= 2mathbf{G} + frac{pi}{3} ln (2 + sqrt{3}) + 2int_0^{frac{pi}{4}} ln (tan theta) , dtheta - 2 int_0^{frac{pi}{12}} ln (tan theta) , dtheta.
            end{align}

            For the first of the integrals we have
            $$int_0^{frac{pi}{4}} ln (tan x) , dx = -mathbf{G}.$$
            While for the second of the integrals we have
            $$int_0^{frac{pi}{12}} ln (tan x) , dx = -frac{2}{3} mathbf{G}.$$
            So finally
            $$I(2) = 2mathbf{G} + frac{pi}{3} ln (2 + sqrt{3}) - 2mathbf{G} + frac{4}{3} mathbf{G},$$
            or
            $$int_0^{frac{pi}{2}} operatorname{arcsinh} (2 tan x) , dx = frac{pi}{3} ln (2 + sqrt{3}) + frac{4}{3} mathbf{G},$$
            as announced.






            share|cite|improve this answer









            $endgroup$



            Here is another variation along a theme which like @DavidG approach uses Feynman's trick of differentiating under the integral sign.



            Let
            $$I(a) = int_0^{frac{pi}{2}} operatorname{arcsinh} (a tan x) , dx, qquad a > 1.$$
            We are required to find $I(2)$. We start by finding $I(1)$ first, which will be needed later on.



            For $a = 1$ we have
            begin{align}
            I(1) &= int_0^{frac{pi}{2}} operatorname{arcsinh} (tan x) , dx\
            &= int_0^{frac{pi}{2}} ln left (frac{1 + sin x}{cos x} right ) , dx\
            &= int_0^{frac{pi}{2}} ln (1 + sin x) , dx - int_0^{frac{pi}{2}} ln (cos x) , dx.
            end{align}

            Now the first of these integrals can be found, for example, by rewriting it as
            begin{align}
            int_0^{frac{pi}{2}} ln (1 + sin x) , dx &= int_0^{frac{pi}{2}} ln (1 + cos x) , dx\
            &= int_0^{frac{pi}{2}} ln left (frac{1}{2} cos^2 frac{x}{2} right ) , dx\
            &= frac{pi}{2} ln 2 + 4 int_0^{frac{pi}{4}} ln (cos x) , dx
            end{align}

            and then employing the Fourier series representation for $ln (cos x)$ found here. The final result is
            $$int_0^{frac{pi}{2}} ln (1 + sin x) , dx = 2 mathbf{G} - frac{pi}{2} ln 2.$$
            Here $mathbf{G}$ is Catalan's constant.



            The second of the integrals is very well known. Here
            $$int_0^{frac{pi}{2}} ln (cos x) , dx = - frac{pi}{2} ln 2.$$
            Thus $I(1) = 2 mathbf{G}$.



            Moving to the main event, by differentiating under the integral sign with respect to $a$ we have
            begin{align}
            I'(a) &= int_0^{frac{pi}{2}} frac{tan x}{sqrt{a^2 tan^2 x + 1}} , dx\
            &= int_0^{frac{pi}{2}} frac{sin x}{sqrt{a^2 - (a^2 - 1) cos^2 x}} , dx.
            end{align}

            Observe the term $(a^2 - 1)$ is positive since $a > 1$. Letting $u = cos x$ one has
            $$I'(a) = int_0^1 frac{du}{sqrt{a^2 - (a^2 - 1) u^2}} = frac{1}{sqrt{a^2 - 1}} sin^{-1} left (frac{sqrt{a^2 - 1}}{a} right ).$$



            As we require $I(2)$ observe that
            $$I(2) - I(1) = int_1^2 I'(a) , da.$$
            Thus
            $$I(2) = 2 mathbf{G} + int_1^2 frac{1}{sqrt{a^2 - 1}} sin^{-1} left (frac{sqrt{a^2 - 1}}{a} right ) , da.$$
            Integrating by parts leads to
            $$I(2) = 2 mathbf{G} + frac{pi}{3} ln (2 + sqrt{3}) - int_1^2 frac{cosh^{-1} a}{a sqrt{a^2 - 1}} , da.$$
            Now let $a = cosh t$. This gives
            $$I(2) = 2mathbf{G} + frac{pi}{3} ln (2 + sqrt{3}) - int_0^{ln(2 + sqrt{3})} frac{t}{cosh t} , dt.$$
            Then let $t = ln y$. This gives
            $$I(2) = 2mathbf{G} + frac{pi}{3} ln (2 + sqrt{3}) - 2int_1^{2 + sqrt{3}} frac{ln y}{1 + y^2} , dy.$$
            Now let $y = tan theta$. Then we have
            $$I(2) = 2mathbf{G} + frac{pi}{3} ln (2 + sqrt{3}) - 2int_{frac{pi}{4}}^{frac{5pi}{12}} ln (tan theta) , dtheta.$$
            Finally, enforcing a substitution of $theta mapsto dfrac{pi}{2} - theta$ leads to
            begin{align}
            I(2) &= 2mathbf{G} + frac{pi}{3} ln (2 + sqrt{3}) + 2 int_{frac{pi}{12}}^{frac{pi}{4}} ln (tan theta) , dtheta\
            &= 2mathbf{G} + frac{pi}{3} ln (2 + sqrt{3}) + 2int_0^{frac{pi}{4}} ln (tan theta) , dtheta - 2 int_0^{frac{pi}{12}} ln (tan theta) , dtheta.
            end{align}

            For the first of the integrals we have
            $$int_0^{frac{pi}{4}} ln (tan x) , dx = -mathbf{G}.$$
            While for the second of the integrals we have
            $$int_0^{frac{pi}{12}} ln (tan x) , dx = -frac{2}{3} mathbf{G}.$$
            So finally
            $$I(2) = 2mathbf{G} + frac{pi}{3} ln (2 + sqrt{3}) - 2mathbf{G} + frac{4}{3} mathbf{G},$$
            or
            $$int_0^{frac{pi}{2}} operatorname{arcsinh} (2 tan x) , dx = frac{pi}{3} ln (2 + sqrt{3}) + frac{4}{3} mathbf{G},$$
            as announced.







            share|cite|improve this answer












            share|cite|improve this answer



            share|cite|improve this answer










            answered Jan 23 at 2:51









            omegadotomegadot

            6,3972829




            6,3972829








            • 1




              $begingroup$
              begin{align}A&=int_0^{frac{pi}{4}}ln(cos x),dx\ B&=int_0^{frac{pi}{4}}ln(sin x),dx\ B-A&=int_0^{frac{pi}{4}}ln(tan x),dx\ &=-text{G}\ A+B&=int_0^{frac{pi}{4}}ln(sin xcos x),dx\ &=int_0^{frac{pi}{4}}lnleft(frac{sin(2x)}{2}right),dx\ &=frac{1}{2}int_0^{frac{pi}{2}}lnleft(frac{sin x}{2}right),dx\ &=frac{1}{2}int_0^{frac{pi}{2}}lnleft(sin xright),dx-frac{pi}{4}ln 2\ &=-frac{1}{2}piln 2\ A&=frac{1}{2}left(A+B-left(B-Aright)right)\ &=frac{1}{2}text{G}-frac{1}{4}piln 2 end{align}
              $endgroup$
              – FDP
              Jan 23 at 13:44










            • $begingroup$
              Yes, very nice.
              $endgroup$
              – omegadot
              Jan 24 at 1:29














            • 1




              $begingroup$
              begin{align}A&=int_0^{frac{pi}{4}}ln(cos x),dx\ B&=int_0^{frac{pi}{4}}ln(sin x),dx\ B-A&=int_0^{frac{pi}{4}}ln(tan x),dx\ &=-text{G}\ A+B&=int_0^{frac{pi}{4}}ln(sin xcos x),dx\ &=int_0^{frac{pi}{4}}lnleft(frac{sin(2x)}{2}right),dx\ &=frac{1}{2}int_0^{frac{pi}{2}}lnleft(frac{sin x}{2}right),dx\ &=frac{1}{2}int_0^{frac{pi}{2}}lnleft(sin xright),dx-frac{pi}{4}ln 2\ &=-frac{1}{2}piln 2\ A&=frac{1}{2}left(A+B-left(B-Aright)right)\ &=frac{1}{2}text{G}-frac{1}{4}piln 2 end{align}
              $endgroup$
              – FDP
              Jan 23 at 13:44










            • $begingroup$
              Yes, very nice.
              $endgroup$
              – omegadot
              Jan 24 at 1:29








            1




            1




            $begingroup$
            begin{align}A&=int_0^{frac{pi}{4}}ln(cos x),dx\ B&=int_0^{frac{pi}{4}}ln(sin x),dx\ B-A&=int_0^{frac{pi}{4}}ln(tan x),dx\ &=-text{G}\ A+B&=int_0^{frac{pi}{4}}ln(sin xcos x),dx\ &=int_0^{frac{pi}{4}}lnleft(frac{sin(2x)}{2}right),dx\ &=frac{1}{2}int_0^{frac{pi}{2}}lnleft(frac{sin x}{2}right),dx\ &=frac{1}{2}int_0^{frac{pi}{2}}lnleft(sin xright),dx-frac{pi}{4}ln 2\ &=-frac{1}{2}piln 2\ A&=frac{1}{2}left(A+B-left(B-Aright)right)\ &=frac{1}{2}text{G}-frac{1}{4}piln 2 end{align}
            $endgroup$
            – FDP
            Jan 23 at 13:44




            $begingroup$
            begin{align}A&=int_0^{frac{pi}{4}}ln(cos x),dx\ B&=int_0^{frac{pi}{4}}ln(sin x),dx\ B-A&=int_0^{frac{pi}{4}}ln(tan x),dx\ &=-text{G}\ A+B&=int_0^{frac{pi}{4}}ln(sin xcos x),dx\ &=int_0^{frac{pi}{4}}lnleft(frac{sin(2x)}{2}right),dx\ &=frac{1}{2}int_0^{frac{pi}{2}}lnleft(frac{sin x}{2}right),dx\ &=frac{1}{2}int_0^{frac{pi}{2}}lnleft(sin xright),dx-frac{pi}{4}ln 2\ &=-frac{1}{2}piln 2\ A&=frac{1}{2}left(A+B-left(B-Aright)right)\ &=frac{1}{2}text{G}-frac{1}{4}piln 2 end{align}
            $endgroup$
            – FDP
            Jan 23 at 13:44












            $begingroup$
            Yes, very nice.
            $endgroup$
            – omegadot
            Jan 24 at 1:29




            $begingroup$
            Yes, very nice.
            $endgroup$
            – omegadot
            Jan 24 at 1:29











            3












            $begingroup$

            Sorry in a rush, so only a PARTIAL SOLUTION:



            Here I will employ Feynman's Trick:
            begin{equation}
            I = int_0^{infty}frac{operatorname{arcsinh(2x)}}{1 + x^2}:dx
            end{equation}



            Let
            begin{equation}
            J(t) = int_0^{infty}frac{operatorname{arcsinh(tx)}}{1 + x^2}:dx
            end{equation}



            We observe that $J(2) = I$ and $J(0) = 0$. Using Leibniz's Integral rule we differentiate with respect to '$t$':



            begin{equation}
            J'(t) = int_0^{infty}frac{x}{sqrt{1 + t^2x^2}}frac{1}{1 + x^2}:dx = left[frac{1}{sqrt{t^2 - 1}} cdot arctanleft(sqrt{frac{1 + t^2x^2}{t^2 - 1}} right) right]_0^{infty} = frac{1}{sqrt{t^2 - 1}}left[frac{pi}{2} - arctanleft(frac{1}{sqrt{t^2 -1}} right) right]
            end{equation}



            Thus,



            begin{align}
            J(t) &= int frac{1}{sqrt{t^2 - 1}}left[frac{pi}{2} - arctanleft(frac{x}{sqrt{t^2 -1}} right) right]:dt \
            &= int frac{pi}{2}cdot frac{1}{sqrt{t^2 - 1}}:dt - int frac{1}{sqrt{t^2 - 1}}arctanleft(frac{1}{sqrt{t^2 -1}} right):dt = I_1 - I_2
            end{align}



            For $I_1$:



            begin{equation}
            I_1 = int frac{pi}{2}cdot frac{1}{sqrt{t^2 - 1}}:dt = frac{pi}{2}lnleft| sqrt{t^2 - 1} + tright| + C_1
            end{equation}



            Where $C_1$ is the constant of integration. Note that $I_1(2) = lnleft|2 + sqrt{3} right|$



            For $I_2$:



            begin{equation}
            I_2 = int frac{1}{sqrt{t^2 - 1}}arctanleft(frac{1}{sqrt{t^2 -1}} right):dt
            end{equation}



            Unfortunately this is not so easy to evaluate. I will first make the substitution $u = frac{1}{sqrt{t^2 - 1}}$:



            begin{align}
            I_2 &= int frac{1}{sqrt{t^2 - 1}}arctanleft(frac{1}{sqrt{t^2 -1}} right):dt = int u cdot arctan(u) cdot frac{-u^4}{sqrt{1 +u^2}}:du\
            & = - int frac{-u^5}{sqrt{1 +u^2}} cdot arctan(u) :du
            end{align}






            share|cite|improve this answer











            $endgroup$













            • $begingroup$
              Will post up full solution later.
              $endgroup$
              – DavidG
              Jan 22 at 10:08
















            3












            $begingroup$

            Sorry in a rush, so only a PARTIAL SOLUTION:



            Here I will employ Feynman's Trick:
            begin{equation}
            I = int_0^{infty}frac{operatorname{arcsinh(2x)}}{1 + x^2}:dx
            end{equation}



            Let
            begin{equation}
            J(t) = int_0^{infty}frac{operatorname{arcsinh(tx)}}{1 + x^2}:dx
            end{equation}



            We observe that $J(2) = I$ and $J(0) = 0$. Using Leibniz's Integral rule we differentiate with respect to '$t$':



            begin{equation}
            J'(t) = int_0^{infty}frac{x}{sqrt{1 + t^2x^2}}frac{1}{1 + x^2}:dx = left[frac{1}{sqrt{t^2 - 1}} cdot arctanleft(sqrt{frac{1 + t^2x^2}{t^2 - 1}} right) right]_0^{infty} = frac{1}{sqrt{t^2 - 1}}left[frac{pi}{2} - arctanleft(frac{1}{sqrt{t^2 -1}} right) right]
            end{equation}



            Thus,



            begin{align}
            J(t) &= int frac{1}{sqrt{t^2 - 1}}left[frac{pi}{2} - arctanleft(frac{x}{sqrt{t^2 -1}} right) right]:dt \
            &= int frac{pi}{2}cdot frac{1}{sqrt{t^2 - 1}}:dt - int frac{1}{sqrt{t^2 - 1}}arctanleft(frac{1}{sqrt{t^2 -1}} right):dt = I_1 - I_2
            end{align}



            For $I_1$:



            begin{equation}
            I_1 = int frac{pi}{2}cdot frac{1}{sqrt{t^2 - 1}}:dt = frac{pi}{2}lnleft| sqrt{t^2 - 1} + tright| + C_1
            end{equation}



            Where $C_1$ is the constant of integration. Note that $I_1(2) = lnleft|2 + sqrt{3} right|$



            For $I_2$:



            begin{equation}
            I_2 = int frac{1}{sqrt{t^2 - 1}}arctanleft(frac{1}{sqrt{t^2 -1}} right):dt
            end{equation}



            Unfortunately this is not so easy to evaluate. I will first make the substitution $u = frac{1}{sqrt{t^2 - 1}}$:



            begin{align}
            I_2 &= int frac{1}{sqrt{t^2 - 1}}arctanleft(frac{1}{sqrt{t^2 -1}} right):dt = int u cdot arctan(u) cdot frac{-u^4}{sqrt{1 +u^2}}:du\
            & = - int frac{-u^5}{sqrt{1 +u^2}} cdot arctan(u) :du
            end{align}






            share|cite|improve this answer











            $endgroup$













            • $begingroup$
              Will post up full solution later.
              $endgroup$
              – DavidG
              Jan 22 at 10:08














            3












            3








            3





            $begingroup$

            Sorry in a rush, so only a PARTIAL SOLUTION:



            Here I will employ Feynman's Trick:
            begin{equation}
            I = int_0^{infty}frac{operatorname{arcsinh(2x)}}{1 + x^2}:dx
            end{equation}



            Let
            begin{equation}
            J(t) = int_0^{infty}frac{operatorname{arcsinh(tx)}}{1 + x^2}:dx
            end{equation}



            We observe that $J(2) = I$ and $J(0) = 0$. Using Leibniz's Integral rule we differentiate with respect to '$t$':



            begin{equation}
            J'(t) = int_0^{infty}frac{x}{sqrt{1 + t^2x^2}}frac{1}{1 + x^2}:dx = left[frac{1}{sqrt{t^2 - 1}} cdot arctanleft(sqrt{frac{1 + t^2x^2}{t^2 - 1}} right) right]_0^{infty} = frac{1}{sqrt{t^2 - 1}}left[frac{pi}{2} - arctanleft(frac{1}{sqrt{t^2 -1}} right) right]
            end{equation}



            Thus,



            begin{align}
            J(t) &= int frac{1}{sqrt{t^2 - 1}}left[frac{pi}{2} - arctanleft(frac{x}{sqrt{t^2 -1}} right) right]:dt \
            &= int frac{pi}{2}cdot frac{1}{sqrt{t^2 - 1}}:dt - int frac{1}{sqrt{t^2 - 1}}arctanleft(frac{1}{sqrt{t^2 -1}} right):dt = I_1 - I_2
            end{align}



            For $I_1$:



            begin{equation}
            I_1 = int frac{pi}{2}cdot frac{1}{sqrt{t^2 - 1}}:dt = frac{pi}{2}lnleft| sqrt{t^2 - 1} + tright| + C_1
            end{equation}



            Where $C_1$ is the constant of integration. Note that $I_1(2) = lnleft|2 + sqrt{3} right|$



            For $I_2$:



            begin{equation}
            I_2 = int frac{1}{sqrt{t^2 - 1}}arctanleft(frac{1}{sqrt{t^2 -1}} right):dt
            end{equation}



            Unfortunately this is not so easy to evaluate. I will first make the substitution $u = frac{1}{sqrt{t^2 - 1}}$:



            begin{align}
            I_2 &= int frac{1}{sqrt{t^2 - 1}}arctanleft(frac{1}{sqrt{t^2 -1}} right):dt = int u cdot arctan(u) cdot frac{-u^4}{sqrt{1 +u^2}}:du\
            & = - int frac{-u^5}{sqrt{1 +u^2}} cdot arctan(u) :du
            end{align}






            share|cite|improve this answer











            $endgroup$



            Sorry in a rush, so only a PARTIAL SOLUTION:



            Here I will employ Feynman's Trick:
            begin{equation}
            I = int_0^{infty}frac{operatorname{arcsinh(2x)}}{1 + x^2}:dx
            end{equation}



            Let
            begin{equation}
            J(t) = int_0^{infty}frac{operatorname{arcsinh(tx)}}{1 + x^2}:dx
            end{equation}



            We observe that $J(2) = I$ and $J(0) = 0$. Using Leibniz's Integral rule we differentiate with respect to '$t$':



            begin{equation}
            J'(t) = int_0^{infty}frac{x}{sqrt{1 + t^2x^2}}frac{1}{1 + x^2}:dx = left[frac{1}{sqrt{t^2 - 1}} cdot arctanleft(sqrt{frac{1 + t^2x^2}{t^2 - 1}} right) right]_0^{infty} = frac{1}{sqrt{t^2 - 1}}left[frac{pi}{2} - arctanleft(frac{1}{sqrt{t^2 -1}} right) right]
            end{equation}



            Thus,



            begin{align}
            J(t) &= int frac{1}{sqrt{t^2 - 1}}left[frac{pi}{2} - arctanleft(frac{x}{sqrt{t^2 -1}} right) right]:dt \
            &= int frac{pi}{2}cdot frac{1}{sqrt{t^2 - 1}}:dt - int frac{1}{sqrt{t^2 - 1}}arctanleft(frac{1}{sqrt{t^2 -1}} right):dt = I_1 - I_2
            end{align}



            For $I_1$:



            begin{equation}
            I_1 = int frac{pi}{2}cdot frac{1}{sqrt{t^2 - 1}}:dt = frac{pi}{2}lnleft| sqrt{t^2 - 1} + tright| + C_1
            end{equation}



            Where $C_1$ is the constant of integration. Note that $I_1(2) = lnleft|2 + sqrt{3} right|$



            For $I_2$:



            begin{equation}
            I_2 = int frac{1}{sqrt{t^2 - 1}}arctanleft(frac{1}{sqrt{t^2 -1}} right):dt
            end{equation}



            Unfortunately this is not so easy to evaluate. I will first make the substitution $u = frac{1}{sqrt{t^2 - 1}}$:



            begin{align}
            I_2 &= int frac{1}{sqrt{t^2 - 1}}arctanleft(frac{1}{sqrt{t^2 -1}} right):dt = int u cdot arctan(u) cdot frac{-u^4}{sqrt{1 +u^2}}:du\
            & = - int frac{-u^5}{sqrt{1 +u^2}} cdot arctan(u) :du
            end{align}







            share|cite|improve this answer














            share|cite|improve this answer



            share|cite|improve this answer








            edited Jan 23 at 6:07

























            answered Jan 22 at 10:07









            DavidGDavidG

            2,5061726




            2,5061726












            • $begingroup$
              Will post up full solution later.
              $endgroup$
              – DavidG
              Jan 22 at 10:08


















            • $begingroup$
              Will post up full solution later.
              $endgroup$
              – DavidG
              Jan 22 at 10:08
















            $begingroup$
            Will post up full solution later.
            $endgroup$
            – DavidG
            Jan 22 at 10:08




            $begingroup$
            Will post up full solution later.
            $endgroup$
            – DavidG
            Jan 22 at 10:08











            3












            $begingroup$

            $$
            begin{align}newcommand{arcsinh}{operatorname{arcsinh}}
            int_0^inftyfrac{arcsinh(x)}{1+x^2},mathrm{d}x
            &=int_0^inftyfrac{x,mathrm{d}x}{cosh(x)}tag{1a}\
            &=int_0^inftyfrac{2x,mathrm{d}x}{e^x+e^{-x}}tag{1b}\
            &=2sum_{k=0}^infty(-1)^kint_0^infty x,e^{-(2k+1)x},mathrm{d}xtag{1c}\ &=2sum_{k=0}^inftyfrac{(-1)^k}{(2k+1)^2}tag{1d}\[6pt]
            &=2mathrm{G}tag{1e}
            end{align}
            $$

            Explanation:
            $text{(1a)}$: substitute $xmapstosinh(x)$
            $text{(1b)}$: write $cosh(x)$ as exponentials
            $text{(1c)}$: expand $left(e^x+e^{-x}right)^{-1}$ as a power series
            $text{(1d)}$: evaluate the integral
            $text{(1e)}$: use the definition of Catalan's Constant
            $$
            begin{align}
            frac{mathrm{d}}{mathrm{d}a}int_0^inftyfrac{arcsinh(ax)}{1+x^2},mathrm{d}x
            &=int_0^inftyfrac{x}{1+x^2}frac{mathrm{d}x}{sqrt{1+a^2x^2}}tag{2a}\
            &=int_0^inftyfrac{x}{a^2+x^2}frac{mathrm{d}x}{sqrt{1+x^2}}tag{2b}\
            &=int_0^{pi/2}frac{mathrm{d}sec(x)}{a^2+tan^2(x)}tag{2c}\
            &=int_1^inftyfrac{mathrm{d}x}{x^2+left(a^2-1right)}tag{2d}\
            &=frac{arctanleft(sqrt{a^2-1}right)}{sqrt{a^2-1}}tag{2e}
            end{align}
            $$

            Explanation:
            $text{(2a)}$: $frac{mathrm{d}}{mathrm{d}a}arcsinh(ax)=frac{x}{sqrt{1+a^2x^2}}$
            $text{(2b)}$: substitute $xmapsto x/a$
            $text{(2c)}$: substitute $xmapstotan(x)$
            $text{(2d)}$: substitute $sec(x)mapsto x$
            $text{(2e)}$: arctan integral



            Therefore,
            $$
            begin{align}newcommand{Li}{operatorname{Li}}
            int_0^inftyfrac{arcsinh(2x)}{1+x^2},mathrm{d}x
            &=2mathrm{G}+int_1^2frac{arctanleft(sqrt{a^2-1}right)}{sqrt{a^2-1}},mathrm{d}atag{3a}\
            &=2mathrm{G}+int_0^{pi/3}frac{a}{cos(a)},mathrm{d}atag{3b}\
            &=2mathrm{G}+2int_0^{pi/3}frac{a}{e^{ia}+e^{-ia}},mathrm{d}atag{3c}\
            &=2mathrm{G}+2sum_{k=0}^infty(-1)^kint_0^{pi/3}ae^{-i(2k+1)a},mathrm{d}atag{3d}\
            &=2mathrm{G}+2sum_{k=0}^inftyfrac{(-1)^k}{(2k+1)^2}int_0^{(2k+1)pi/3}ae^{-ia},mathrm{d}atag{3e}\
            &=2mathrm{G}+2sum_{k=0}^inftyfrac{(-1)^k}{(2k+1)^2}left[(1+i(2k+1)pi/3)e^{-i(2k+1)pi/3}-1right]tag{3f}\
            &=2mathrm{G}+fracpi3logleft(frac{2+sqrt3+i}{2-sqrt3-i}right)+2sum_{k=0}^inftyfrac{(-1)^k}{(2k+1)^2}left[e^{-i(2k+1)pi/3}-1right]tag{3g}\
            &=2mathrm{G}+fracpi3logleft(left(2+sqrt3right)iright)
            +sum_{k=0}^infty(-1)^kscriptsizeleft[frac{-1-isqrt3}{(6k+1)^2}-frac{-4}{(6k+3)^2}+frac{-1+isqrt3}{(6k+5)^2}right]tag{3h}\
            &=frac43mathrm{G}+fracpi3logleft(2+sqrt3right)+ileft[frac{pi^2}6-sqrt3sum_{k=0}^infty(-1)^kscriptsizeleft(frac1{(6k+1)^2}-frac1{(6k+5)^2}right)right]tag{3i}\
            &=frac43mathrm{G}+fracpi3logleft(2+sqrt3right)+ileft[frac{pi^2}6-frac{sqrt3}{36}sum_{kinmathbb{Z}}frac{(-1)^k}{left(k+frac16right)^2}right]tag{3j}\
            &=bbox[5px,border:2px solid #C0A000]{frac43mathrm{G}+fracpi3logleft(2+sqrt3right)}tag{3k}
            end{align}
            $$

            Explanation:
            $text{(3a)}$: apply $(1)$ and $(2)$
            $text{(3b)}$: substitute $amapstosec(a)$
            $text{(3c)}$: write $cos(x)$ as exponentials
            $text{(3d)}$: expand $left(e^{ix}+e^{-ix}right)^{-1}$ as a power series
            $text{(3e)}$: substitute $amapsto a/(2k+1)$
            $text{(3f)}$: integrate
            $text{(3g)}$: recognize the series for $arctan(x)=frac1{2i}logleft(frac{1+ix}{1-ix}right)$
            $text{(3h)}$: evaluate the exponentials
            $text{(3i)}$: separate the real and imaginary parts
            $text{(3j)}$: rewrite the sum
            $text{(3k)}$: $sum_{kinmathbb{Z}}frac{(-1)^k}{(k+x)^2}=pi^2cot(pi x)csc(pi x)$






            share|cite|improve this answer











            $endgroup$













            • $begingroup$
              Seems like there should be a ^2 in the denominator of the sum of the explanation (3k)
              $endgroup$
              – Kemono Chen
              Jan 24 at 9:03










            • $begingroup$
              @KemonoChen: Indeed. I put the parentheses there because I meant to square the denominator. Thanks. While it's true that we can just take the real part of the right side since the quantity is real, I wanted to make sure I hadn't made a mistake and verify that the imaginary part was $0$.
              $endgroup$
              – robjohn
              Jan 24 at 9:25


















            3












            $begingroup$

            $$
            begin{align}newcommand{arcsinh}{operatorname{arcsinh}}
            int_0^inftyfrac{arcsinh(x)}{1+x^2},mathrm{d}x
            &=int_0^inftyfrac{x,mathrm{d}x}{cosh(x)}tag{1a}\
            &=int_0^inftyfrac{2x,mathrm{d}x}{e^x+e^{-x}}tag{1b}\
            &=2sum_{k=0}^infty(-1)^kint_0^infty x,e^{-(2k+1)x},mathrm{d}xtag{1c}\ &=2sum_{k=0}^inftyfrac{(-1)^k}{(2k+1)^2}tag{1d}\[6pt]
            &=2mathrm{G}tag{1e}
            end{align}
            $$

            Explanation:
            $text{(1a)}$: substitute $xmapstosinh(x)$
            $text{(1b)}$: write $cosh(x)$ as exponentials
            $text{(1c)}$: expand $left(e^x+e^{-x}right)^{-1}$ as a power series
            $text{(1d)}$: evaluate the integral
            $text{(1e)}$: use the definition of Catalan's Constant
            $$
            begin{align}
            frac{mathrm{d}}{mathrm{d}a}int_0^inftyfrac{arcsinh(ax)}{1+x^2},mathrm{d}x
            &=int_0^inftyfrac{x}{1+x^2}frac{mathrm{d}x}{sqrt{1+a^2x^2}}tag{2a}\
            &=int_0^inftyfrac{x}{a^2+x^2}frac{mathrm{d}x}{sqrt{1+x^2}}tag{2b}\
            &=int_0^{pi/2}frac{mathrm{d}sec(x)}{a^2+tan^2(x)}tag{2c}\
            &=int_1^inftyfrac{mathrm{d}x}{x^2+left(a^2-1right)}tag{2d}\
            &=frac{arctanleft(sqrt{a^2-1}right)}{sqrt{a^2-1}}tag{2e}
            end{align}
            $$

            Explanation:
            $text{(2a)}$: $frac{mathrm{d}}{mathrm{d}a}arcsinh(ax)=frac{x}{sqrt{1+a^2x^2}}$
            $text{(2b)}$: substitute $xmapsto x/a$
            $text{(2c)}$: substitute $xmapstotan(x)$
            $text{(2d)}$: substitute $sec(x)mapsto x$
            $text{(2e)}$: arctan integral



            Therefore,
            $$
            begin{align}newcommand{Li}{operatorname{Li}}
            int_0^inftyfrac{arcsinh(2x)}{1+x^2},mathrm{d}x
            &=2mathrm{G}+int_1^2frac{arctanleft(sqrt{a^2-1}right)}{sqrt{a^2-1}},mathrm{d}atag{3a}\
            &=2mathrm{G}+int_0^{pi/3}frac{a}{cos(a)},mathrm{d}atag{3b}\
            &=2mathrm{G}+2int_0^{pi/3}frac{a}{e^{ia}+e^{-ia}},mathrm{d}atag{3c}\
            &=2mathrm{G}+2sum_{k=0}^infty(-1)^kint_0^{pi/3}ae^{-i(2k+1)a},mathrm{d}atag{3d}\
            &=2mathrm{G}+2sum_{k=0}^inftyfrac{(-1)^k}{(2k+1)^2}int_0^{(2k+1)pi/3}ae^{-ia},mathrm{d}atag{3e}\
            &=2mathrm{G}+2sum_{k=0}^inftyfrac{(-1)^k}{(2k+1)^2}left[(1+i(2k+1)pi/3)e^{-i(2k+1)pi/3}-1right]tag{3f}\
            &=2mathrm{G}+fracpi3logleft(frac{2+sqrt3+i}{2-sqrt3-i}right)+2sum_{k=0}^inftyfrac{(-1)^k}{(2k+1)^2}left[e^{-i(2k+1)pi/3}-1right]tag{3g}\
            &=2mathrm{G}+fracpi3logleft(left(2+sqrt3right)iright)
            +sum_{k=0}^infty(-1)^kscriptsizeleft[frac{-1-isqrt3}{(6k+1)^2}-frac{-4}{(6k+3)^2}+frac{-1+isqrt3}{(6k+5)^2}right]tag{3h}\
            &=frac43mathrm{G}+fracpi3logleft(2+sqrt3right)+ileft[frac{pi^2}6-sqrt3sum_{k=0}^infty(-1)^kscriptsizeleft(frac1{(6k+1)^2}-frac1{(6k+5)^2}right)right]tag{3i}\
            &=frac43mathrm{G}+fracpi3logleft(2+sqrt3right)+ileft[frac{pi^2}6-frac{sqrt3}{36}sum_{kinmathbb{Z}}frac{(-1)^k}{left(k+frac16right)^2}right]tag{3j}\
            &=bbox[5px,border:2px solid #C0A000]{frac43mathrm{G}+fracpi3logleft(2+sqrt3right)}tag{3k}
            end{align}
            $$

            Explanation:
            $text{(3a)}$: apply $(1)$ and $(2)$
            $text{(3b)}$: substitute $amapstosec(a)$
            $text{(3c)}$: write $cos(x)$ as exponentials
            $text{(3d)}$: expand $left(e^{ix}+e^{-ix}right)^{-1}$ as a power series
            $text{(3e)}$: substitute $amapsto a/(2k+1)$
            $text{(3f)}$: integrate
            $text{(3g)}$: recognize the series for $arctan(x)=frac1{2i}logleft(frac{1+ix}{1-ix}right)$
            $text{(3h)}$: evaluate the exponentials
            $text{(3i)}$: separate the real and imaginary parts
            $text{(3j)}$: rewrite the sum
            $text{(3k)}$: $sum_{kinmathbb{Z}}frac{(-1)^k}{(k+x)^2}=pi^2cot(pi x)csc(pi x)$






            share|cite|improve this answer











            $endgroup$













            • $begingroup$
              Seems like there should be a ^2 in the denominator of the sum of the explanation (3k)
              $endgroup$
              – Kemono Chen
              Jan 24 at 9:03










            • $begingroup$
              @KemonoChen: Indeed. I put the parentheses there because I meant to square the denominator. Thanks. While it's true that we can just take the real part of the right side since the quantity is real, I wanted to make sure I hadn't made a mistake and verify that the imaginary part was $0$.
              $endgroup$
              – robjohn
              Jan 24 at 9:25
















            3












            3








            3





            $begingroup$

            $$
            begin{align}newcommand{arcsinh}{operatorname{arcsinh}}
            int_0^inftyfrac{arcsinh(x)}{1+x^2},mathrm{d}x
            &=int_0^inftyfrac{x,mathrm{d}x}{cosh(x)}tag{1a}\
            &=int_0^inftyfrac{2x,mathrm{d}x}{e^x+e^{-x}}tag{1b}\
            &=2sum_{k=0}^infty(-1)^kint_0^infty x,e^{-(2k+1)x},mathrm{d}xtag{1c}\ &=2sum_{k=0}^inftyfrac{(-1)^k}{(2k+1)^2}tag{1d}\[6pt]
            &=2mathrm{G}tag{1e}
            end{align}
            $$

            Explanation:
            $text{(1a)}$: substitute $xmapstosinh(x)$
            $text{(1b)}$: write $cosh(x)$ as exponentials
            $text{(1c)}$: expand $left(e^x+e^{-x}right)^{-1}$ as a power series
            $text{(1d)}$: evaluate the integral
            $text{(1e)}$: use the definition of Catalan's Constant
            $$
            begin{align}
            frac{mathrm{d}}{mathrm{d}a}int_0^inftyfrac{arcsinh(ax)}{1+x^2},mathrm{d}x
            &=int_0^inftyfrac{x}{1+x^2}frac{mathrm{d}x}{sqrt{1+a^2x^2}}tag{2a}\
            &=int_0^inftyfrac{x}{a^2+x^2}frac{mathrm{d}x}{sqrt{1+x^2}}tag{2b}\
            &=int_0^{pi/2}frac{mathrm{d}sec(x)}{a^2+tan^2(x)}tag{2c}\
            &=int_1^inftyfrac{mathrm{d}x}{x^2+left(a^2-1right)}tag{2d}\
            &=frac{arctanleft(sqrt{a^2-1}right)}{sqrt{a^2-1}}tag{2e}
            end{align}
            $$

            Explanation:
            $text{(2a)}$: $frac{mathrm{d}}{mathrm{d}a}arcsinh(ax)=frac{x}{sqrt{1+a^2x^2}}$
            $text{(2b)}$: substitute $xmapsto x/a$
            $text{(2c)}$: substitute $xmapstotan(x)$
            $text{(2d)}$: substitute $sec(x)mapsto x$
            $text{(2e)}$: arctan integral



            Therefore,
            $$
            begin{align}newcommand{Li}{operatorname{Li}}
            int_0^inftyfrac{arcsinh(2x)}{1+x^2},mathrm{d}x
            &=2mathrm{G}+int_1^2frac{arctanleft(sqrt{a^2-1}right)}{sqrt{a^2-1}},mathrm{d}atag{3a}\
            &=2mathrm{G}+int_0^{pi/3}frac{a}{cos(a)},mathrm{d}atag{3b}\
            &=2mathrm{G}+2int_0^{pi/3}frac{a}{e^{ia}+e^{-ia}},mathrm{d}atag{3c}\
            &=2mathrm{G}+2sum_{k=0}^infty(-1)^kint_0^{pi/3}ae^{-i(2k+1)a},mathrm{d}atag{3d}\
            &=2mathrm{G}+2sum_{k=0}^inftyfrac{(-1)^k}{(2k+1)^2}int_0^{(2k+1)pi/3}ae^{-ia},mathrm{d}atag{3e}\
            &=2mathrm{G}+2sum_{k=0}^inftyfrac{(-1)^k}{(2k+1)^2}left[(1+i(2k+1)pi/3)e^{-i(2k+1)pi/3}-1right]tag{3f}\
            &=2mathrm{G}+fracpi3logleft(frac{2+sqrt3+i}{2-sqrt3-i}right)+2sum_{k=0}^inftyfrac{(-1)^k}{(2k+1)^2}left[e^{-i(2k+1)pi/3}-1right]tag{3g}\
            &=2mathrm{G}+fracpi3logleft(left(2+sqrt3right)iright)
            +sum_{k=0}^infty(-1)^kscriptsizeleft[frac{-1-isqrt3}{(6k+1)^2}-frac{-4}{(6k+3)^2}+frac{-1+isqrt3}{(6k+5)^2}right]tag{3h}\
            &=frac43mathrm{G}+fracpi3logleft(2+sqrt3right)+ileft[frac{pi^2}6-sqrt3sum_{k=0}^infty(-1)^kscriptsizeleft(frac1{(6k+1)^2}-frac1{(6k+5)^2}right)right]tag{3i}\
            &=frac43mathrm{G}+fracpi3logleft(2+sqrt3right)+ileft[frac{pi^2}6-frac{sqrt3}{36}sum_{kinmathbb{Z}}frac{(-1)^k}{left(k+frac16right)^2}right]tag{3j}\
            &=bbox[5px,border:2px solid #C0A000]{frac43mathrm{G}+fracpi3logleft(2+sqrt3right)}tag{3k}
            end{align}
            $$

            Explanation:
            $text{(3a)}$: apply $(1)$ and $(2)$
            $text{(3b)}$: substitute $amapstosec(a)$
            $text{(3c)}$: write $cos(x)$ as exponentials
            $text{(3d)}$: expand $left(e^{ix}+e^{-ix}right)^{-1}$ as a power series
            $text{(3e)}$: substitute $amapsto a/(2k+1)$
            $text{(3f)}$: integrate
            $text{(3g)}$: recognize the series for $arctan(x)=frac1{2i}logleft(frac{1+ix}{1-ix}right)$
            $text{(3h)}$: evaluate the exponentials
            $text{(3i)}$: separate the real and imaginary parts
            $text{(3j)}$: rewrite the sum
            $text{(3k)}$: $sum_{kinmathbb{Z}}frac{(-1)^k}{(k+x)^2}=pi^2cot(pi x)csc(pi x)$






            share|cite|improve this answer











            $endgroup$



            $$
            begin{align}newcommand{arcsinh}{operatorname{arcsinh}}
            int_0^inftyfrac{arcsinh(x)}{1+x^2},mathrm{d}x
            &=int_0^inftyfrac{x,mathrm{d}x}{cosh(x)}tag{1a}\
            &=int_0^inftyfrac{2x,mathrm{d}x}{e^x+e^{-x}}tag{1b}\
            &=2sum_{k=0}^infty(-1)^kint_0^infty x,e^{-(2k+1)x},mathrm{d}xtag{1c}\ &=2sum_{k=0}^inftyfrac{(-1)^k}{(2k+1)^2}tag{1d}\[6pt]
            &=2mathrm{G}tag{1e}
            end{align}
            $$

            Explanation:
            $text{(1a)}$: substitute $xmapstosinh(x)$
            $text{(1b)}$: write $cosh(x)$ as exponentials
            $text{(1c)}$: expand $left(e^x+e^{-x}right)^{-1}$ as a power series
            $text{(1d)}$: evaluate the integral
            $text{(1e)}$: use the definition of Catalan's Constant
            $$
            begin{align}
            frac{mathrm{d}}{mathrm{d}a}int_0^inftyfrac{arcsinh(ax)}{1+x^2},mathrm{d}x
            &=int_0^inftyfrac{x}{1+x^2}frac{mathrm{d}x}{sqrt{1+a^2x^2}}tag{2a}\
            &=int_0^inftyfrac{x}{a^2+x^2}frac{mathrm{d}x}{sqrt{1+x^2}}tag{2b}\
            &=int_0^{pi/2}frac{mathrm{d}sec(x)}{a^2+tan^2(x)}tag{2c}\
            &=int_1^inftyfrac{mathrm{d}x}{x^2+left(a^2-1right)}tag{2d}\
            &=frac{arctanleft(sqrt{a^2-1}right)}{sqrt{a^2-1}}tag{2e}
            end{align}
            $$

            Explanation:
            $text{(2a)}$: $frac{mathrm{d}}{mathrm{d}a}arcsinh(ax)=frac{x}{sqrt{1+a^2x^2}}$
            $text{(2b)}$: substitute $xmapsto x/a$
            $text{(2c)}$: substitute $xmapstotan(x)$
            $text{(2d)}$: substitute $sec(x)mapsto x$
            $text{(2e)}$: arctan integral



            Therefore,
            $$
            begin{align}newcommand{Li}{operatorname{Li}}
            int_0^inftyfrac{arcsinh(2x)}{1+x^2},mathrm{d}x
            &=2mathrm{G}+int_1^2frac{arctanleft(sqrt{a^2-1}right)}{sqrt{a^2-1}},mathrm{d}atag{3a}\
            &=2mathrm{G}+int_0^{pi/3}frac{a}{cos(a)},mathrm{d}atag{3b}\
            &=2mathrm{G}+2int_0^{pi/3}frac{a}{e^{ia}+e^{-ia}},mathrm{d}atag{3c}\
            &=2mathrm{G}+2sum_{k=0}^infty(-1)^kint_0^{pi/3}ae^{-i(2k+1)a},mathrm{d}atag{3d}\
            &=2mathrm{G}+2sum_{k=0}^inftyfrac{(-1)^k}{(2k+1)^2}int_0^{(2k+1)pi/3}ae^{-ia},mathrm{d}atag{3e}\
            &=2mathrm{G}+2sum_{k=0}^inftyfrac{(-1)^k}{(2k+1)^2}left[(1+i(2k+1)pi/3)e^{-i(2k+1)pi/3}-1right]tag{3f}\
            &=2mathrm{G}+fracpi3logleft(frac{2+sqrt3+i}{2-sqrt3-i}right)+2sum_{k=0}^inftyfrac{(-1)^k}{(2k+1)^2}left[e^{-i(2k+1)pi/3}-1right]tag{3g}\
            &=2mathrm{G}+fracpi3logleft(left(2+sqrt3right)iright)
            +sum_{k=0}^infty(-1)^kscriptsizeleft[frac{-1-isqrt3}{(6k+1)^2}-frac{-4}{(6k+3)^2}+frac{-1+isqrt3}{(6k+5)^2}right]tag{3h}\
            &=frac43mathrm{G}+fracpi3logleft(2+sqrt3right)+ileft[frac{pi^2}6-sqrt3sum_{k=0}^infty(-1)^kscriptsizeleft(frac1{(6k+1)^2}-frac1{(6k+5)^2}right)right]tag{3i}\
            &=frac43mathrm{G}+fracpi3logleft(2+sqrt3right)+ileft[frac{pi^2}6-frac{sqrt3}{36}sum_{kinmathbb{Z}}frac{(-1)^k}{left(k+frac16right)^2}right]tag{3j}\
            &=bbox[5px,border:2px solid #C0A000]{frac43mathrm{G}+fracpi3logleft(2+sqrt3right)}tag{3k}
            end{align}
            $$

            Explanation:
            $text{(3a)}$: apply $(1)$ and $(2)$
            $text{(3b)}$: substitute $amapstosec(a)$
            $text{(3c)}$: write $cos(x)$ as exponentials
            $text{(3d)}$: expand $left(e^{ix}+e^{-ix}right)^{-1}$ as a power series
            $text{(3e)}$: substitute $amapsto a/(2k+1)$
            $text{(3f)}$: integrate
            $text{(3g)}$: recognize the series for $arctan(x)=frac1{2i}logleft(frac{1+ix}{1-ix}right)$
            $text{(3h)}$: evaluate the exponentials
            $text{(3i)}$: separate the real and imaginary parts
            $text{(3j)}$: rewrite the sum
            $text{(3k)}$: $sum_{kinmathbb{Z}}frac{(-1)^k}{(k+x)^2}=pi^2cot(pi x)csc(pi x)$







            share|cite|improve this answer














            share|cite|improve this answer



            share|cite|improve this answer








            edited Jan 24 at 9:24

























            answered Jan 24 at 8:44









            robjohnrobjohn

            269k27309635




            269k27309635












            • $begingroup$
              Seems like there should be a ^2 in the denominator of the sum of the explanation (3k)
              $endgroup$
              – Kemono Chen
              Jan 24 at 9:03










            • $begingroup$
              @KemonoChen: Indeed. I put the parentheses there because I meant to square the denominator. Thanks. While it's true that we can just take the real part of the right side since the quantity is real, I wanted to make sure I hadn't made a mistake and verify that the imaginary part was $0$.
              $endgroup$
              – robjohn
              Jan 24 at 9:25




















            • $begingroup$
              Seems like there should be a ^2 in the denominator of the sum of the explanation (3k)
              $endgroup$
              – Kemono Chen
              Jan 24 at 9:03










            • $begingroup$
              @KemonoChen: Indeed. I put the parentheses there because I meant to square the denominator. Thanks. While it's true that we can just take the real part of the right side since the quantity is real, I wanted to make sure I hadn't made a mistake and verify that the imaginary part was $0$.
              $endgroup$
              – robjohn
              Jan 24 at 9:25


















            $begingroup$
            Seems like there should be a ^2 in the denominator of the sum of the explanation (3k)
            $endgroup$
            – Kemono Chen
            Jan 24 at 9:03




            $begingroup$
            Seems like there should be a ^2 in the denominator of the sum of the explanation (3k)
            $endgroup$
            – Kemono Chen
            Jan 24 at 9:03












            $begingroup$
            @KemonoChen: Indeed. I put the parentheses there because I meant to square the denominator. Thanks. While it's true that we can just take the real part of the right side since the quantity is real, I wanted to make sure I hadn't made a mistake and verify that the imaginary part was $0$.
            $endgroup$
            – robjohn
            Jan 24 at 9:25






            $begingroup$
            @KemonoChen: Indeed. I put the parentheses there because I meant to square the denominator. Thanks. While it's true that we can just take the real part of the right side since the quantity is real, I wanted to make sure I hadn't made a mistake and verify that the imaginary part was $0$.
            $endgroup$
            – robjohn
            Jan 24 at 9:25













            1












            $begingroup$

            This is not an answer.



            Your premonition seems to be good. Using another CAS,
            $$2int_0^inftyfrac{xcosh( x)}{3+cosh^2(x)},dx=-frac{pi}{4} log left(7-4 sqrt{3}right)-i left(text{Li}_2left(-i left(-2+sqrt{3}right)right)-text{Li}_2left(i
            left(-2+sqrt{3}right)right)right)$$
            Now, ???






            share|cite|improve this answer









            $endgroup$


















              1












              $begingroup$

              This is not an answer.



              Your premonition seems to be good. Using another CAS,
              $$2int_0^inftyfrac{xcosh( x)}{3+cosh^2(x)},dx=-frac{pi}{4} log left(7-4 sqrt{3}right)-i left(text{Li}_2left(-i left(-2+sqrt{3}right)right)-text{Li}_2left(i
              left(-2+sqrt{3}right)right)right)$$
              Now, ???






              share|cite|improve this answer









              $endgroup$
















                1












                1








                1





                $begingroup$

                This is not an answer.



                Your premonition seems to be good. Using another CAS,
                $$2int_0^inftyfrac{xcosh( x)}{3+cosh^2(x)},dx=-frac{pi}{4} log left(7-4 sqrt{3}right)-i left(text{Li}_2left(-i left(-2+sqrt{3}right)right)-text{Li}_2left(i
                left(-2+sqrt{3}right)right)right)$$
                Now, ???






                share|cite|improve this answer









                $endgroup$



                This is not an answer.



                Your premonition seems to be good. Using another CAS,
                $$2int_0^inftyfrac{xcosh( x)}{3+cosh^2(x)},dx=-frac{pi}{4} log left(7-4 sqrt{3}right)-i left(text{Li}_2left(-i left(-2+sqrt{3}right)right)-text{Li}_2left(i
                left(-2+sqrt{3}right)right)right)$$
                Now, ???







                share|cite|improve this answer












                share|cite|improve this answer



                share|cite|improve this answer










                answered Jan 22 at 9:12









                Claude LeiboviciClaude Leibovici

                123k1157135




                123k1157135






























                    draft saved

                    draft discarded




















































                    Thanks for contributing an answer to Mathematics Stack Exchange!


                    • Please be sure to answer the question. Provide details and share your research!

                    But avoid



                    • Asking for help, clarification, or responding to other answers.

                    • Making statements based on opinion; back them up with references or personal experience.


                    Use MathJax to format equations. MathJax reference.


                    To learn more, see our tips on writing great answers.




                    draft saved


                    draft discarded














                    StackExchange.ready(
                    function () {
                    StackExchange.openid.initPostLogin('.new-post-login', 'https%3a%2f%2fmath.stackexchange.com%2fquestions%2f3082875%2fevaluating-int-0-pi-2-operatornamearcsinh2-tan-xdx%23new-answer', 'question_page');
                    }
                    );

                    Post as a guest















                    Required, but never shown





















































                    Required, but never shown














                    Required, but never shown












                    Required, but never shown







                    Required, but never shown

































                    Required, but never shown














                    Required, but never shown












                    Required, but never shown







                    Required, but never shown







                    Popular posts from this blog

                    Can a sorcerer learn a 5th-level spell early by creating spell slots using the Font of Magic feature?

                    ts Property 'filter' does not exist on type '{}'

                    Notepad++ export/extract a list of installed plugins